Categories
Exam Questions Fields Johns Hopkins

Johns Hopkins. Graduate Theory of International Trade Exam. Machlup and Harberger, 1951

 

THE JOHNS HOPKINS UNIVERSITY
Theory of International Trade

Drs. Fritz Machlup and Arnold C. Harberger
May 28, 1951

Answer three questions including I.

I.

Assume that Country A has been importing 1,000,000 tons of X per period over a duty of $2.00 per ton. Domestic production has been 11,000,000 tons per period, the domestic price $6.00 per ton. Pure competition prevails among domestic and foreign producers. Demand and supply conditions remain stable everywhere; the elasticity of domestic demand is -1.0, the elasticity of domestic supply is + 0.5, and the elasticity of the foreign excess supply is infinite.

Now a tariff reduction of $1.00 per ton is granted for imports up to a certain maximum per period; imports in excess of this quota are permitted but subject to the full duty of $2.00. State the effects of the tariff reduction upon domestic price, domestic production, and total imports of X and upon customs revenue, if the tariff quote is

(a) 1,000,000 tons of X per period;
(b) 2,000,000 tons of X per period; and
(c) 4,000,000 tons of X per period.

[Your calculations need not be exact.]

 

II.

Comment on the following statements and discuss whether and under what conditions they may be true or false.

  1. A country cannot gain from imposing an export tax if the foreign demand for its exports is of greater than unit elasticity.
  2. In a country with a relative scarcity of capital the real interest rate will be higher if a general import tariff is imposed than it would be under free trade.
  3. For every situation in which trade is encumbered by tariffs, there exists a situation of unencumbered trade in which all countries involved would be better off.

 

III.

Would you expect “national welfare” in Country A to increase, decrease, or remain the same as a result of

  1. an increase in foreign demand for the export product(s) of country A?
  2. an increase in the foreign excess supply of the product(s) which country A imports?
  3. an improvement in technology in Country A?
  4. a tariff imposed on its imports by Country A?
  5. a tax imposed on its exports by Country A?
  6. a tariff imposed on imports from Country A by country B (the rest of the world)?
  7. a tax Imposed on exports to country A by country B (the rest of the world)?

Give reasons for your answers.

 

IV.

It is generally agreed that the receipt of dollar grants by a country will raise its maximum level of real consumption plus investment. Little has been said in the literature about changes in the relative shares of the increased total which go to the various factors of production. On the basis of the analysis developed in this course, what can be said about the circumstances under which these relative shares are likely to change? Are they likely to change at all? If so, in what direction? If not, why not? In your discussion, assume continuous full employment in all (both granting and receiving) countries, and assume that the grant is used fully by the receiving country.

Source: Johns Hopkins University. Eisenhower Library. Ferdinand Hamburger, Jr. Archives.  Department of Political Economy Series 6. Box 3/1, Folder: “Department of Political Economy. Graduate Exams, 1933-1965.”

Categories
Exam Questions Harvard

Harvard. Semester exams for all economics and one social ethics course, 1893-1894

 

With this post Economics in the Rear-view Mirror adds yet another annual slice of final examinations from Harvard. Over twenty pages of exam questions (with course enrollment figures) for the 1893-94 academic year have been transcribed and are now available to the internet community of historians of economics.  For other years visitors can simply scan or search the chronological catalogue of artifacts. Alternatively using Google search constrained to Economics in the Rear-view Mirror, “harvard economics exams site:irwincollier.com“, will get you links to plenty of Harvard examination postings through the years.

______________________

Enrollment for Philosophy 5.
The Ethics of the Social Questions.
1893-94.

Enrollment.

[Philosophy] 5. Professor Peabody. — The Ethics of the Social Questions. — The questions of Charity, Divorce, the Indians, Temperance, and the various phases of the Labor Question, as problems of practical Ethics. — Lectures, essays, and practical observations. 2 hours.

Total 118: 6 Gr., 56 Se., 23 Ju., 2 So., 12 Others, 19 Divinity.

Source: Harvard University. Report of the President of Harvard College, 1893-94, p. 58.

1893-94.
PHILOSOPHY 5.
THE ETHICS OF THE SOCIAL QUESTIONS.
Mid-Year Examination.

  1. “Political Economy ought to combine with the old question: ‘Will it pay?’ another and higher query: ‘Is it right?’” (C. D. Wright, Political Economy and the Labor Question, p. 17.) The place and value of this view of Political Economy.
  2. Spencer’s formula for conduct, explained and criticized (Data of Ethics, p. 14.)
  3. The Socialist’s view of Charity and the argument which sustains it. Mr. Spencer’s view of Charity and his practical advice. (Principles of Ethics, II. p. 376, ff.)
  4. What does Mr. Charles Booth regard as the “crux” of the Social Problem in East London? (Labour and Life of the People, I. pp. 596 and 162.) Why? The practical remedy proposed by him.
  5. The causes of poverty in East London, as analyzed by Mr. Booth, (I. 147); in their order of importance and the proportion of cases involved.
  6. The Labor Colonies of Germany compared with those of Holland, in method and intention. How far, and under what principle, is such an enterprise applicable to the condition of this country?
  7. Liberalitas” and “Caritas,” — the aim, the service, and the peril of each.
  8. The historical development and the practical rules of the English Poor-Law System.
  9. The Relation of Charity Organization in England to Poor-Law Relief. (Loch, p. 37, ff.); and the objections to Charity Organization. (Loch, p. 97, ff.)
  10. The growth of Charity Organization in the United States, its present extent and its two types (Report, pp. 1-8.) Which type is represented by the London Charity Organization Society? (Loch p. 54.) Which is the sounder principle for this country? Why? Which is the more generally accepted principle? (Appendix of Report, p. 34.)

Source: Harvard University Archives. Harvard University, Mid-year examinations, 1852-1943. Box 3, Volume: Examination Papers, Mid-Year, 1893-94

PHILOSOPHY 5.
THE ETHICS OF THE SOCIAL QUESTIONS.
Final Examination

[Omit one question.]

  1. The authorship and the historical importance of the following phrases:—
    “The value of a thing is independent of opinion and of quantity. To be valuable is to avail towards life.”
    “All commodities are only masses of congealed labor-time.”
    “The high road to a stable sufficiency and comfort among the people is through the medium of their character.”
    “Cash-payment never was or could, except for a few years, be the union-bond of man to man.”
    “Aristocracy of talent.”
    “It is easier to determine what a man ought to have for his work, than what his necessities will compel him to take for it.”
    “Ill-th.”
  2. Compare Carlyle and Ruskin in their attitudes toward the growth of democracy and in their doctrine of social progress.
  3. Compare the view of the “Social Horizon” with that of Naquet as to the effect of collectivism on enterprise and invention. (Social Horizon, pp. 112-151; Naquet, pp. 92-126.)
  4. The Anarchist’s criticism of the Socialist, the Socialist’s criticism of the Anarchist, and the Communist as he is criticised by both.
  5. Is thrift a virtue? Who doubts it? Why?
    Is competition an evil? Who doubts it? Why?
  6. Christian Socialism and its difficulties. The logical and the practical relation of Socialism to Religion.
  7. In the four ideals which are possible to Socialism and Individualism, “the normal relation would be that of cross-correspondence.” (Bosanquet. The Civilization of Christendom, p. 136.) Explain and comment on this statement.
  8. Enumerate and classify the arguments presented in the Course on the ethical aspects of Socialism, with your judgment of the weight of these suggestions.
  9. Compare the plan of profit-sharing in the Paris and Orleans Railway (Sedley Taylor, pp. 77-86) with that adopted by the Chicago, Burlington and Quincy Railroad.
  10. How far are we carried in the argument for abstinence from intoxicating drink by considerations drawn from the “risks of life.” Why?

Source: Harvard University Archives. Harvard University, Examination papers, 1853-2001. Box 2, Volume: Papers Set for Final Examinations in Philosophy, History, Government and Law, Economics, Fine Arts, and Music in Harvard College, June 1894, p. 7.

______________________

Economics 1.
Outlines of Economics.
1893-94.

Enrollment.

[Economics] 1. Professors Taussig and Ashley, Asst. Professor Cummings and Mr. Clow. — Outlines of Economics. — Mill’s Principles of Political Economy. — Lectures on Economic Development, Distribution, Social Questions, and Financial Legislation. 3 hours.

Total 340: 1 Gr., 35 Se., 111 Ju., 136 So., 7 Fr., 50 Others.

Source: Harvard University. Report of the President of Harvard College, 1893-94, p. 61.

1893-94.
ECONOMICS 1.
Mid-Year Examination.

[Arrange your answers strictly in the order of the questions.
One question may be omitted.]

  1. “Let us consider whether, and in what cases, the property of those who live on the interest of what they possess, without being personally engaged in production, can be regarded as capital.” Illustrate by example.
  2. “Capital, though saved, and the result of saving, is nevertheless consumed. The word saving does not imply that what is saved is not consumed, nor even necessarily that its consumption is deferred.” Explain. Who is the consumer? and is the consumption usually deferred?
  3. Are wages likely to be low or high in different occupations because of (1) attractiveness, (2) unpleasantness, of the work? Why?
  4. “This equalizing process, commonly described as the transfer of capital from one employment to another, is not necessarily the slow, onerous, and almost impracticable process which it is often represented to be.” What is the equalizing process? and why is it or is it not slow and onerous?
  5. “Even if there were never any land taken into cultivation for which rent was not paid, it would be true, nevertheless, that there is always some agricultural capital which pays no rent.” Explain, and give the reasons for the statement.
  6. What are the laws of value applicable to: silver bullion, cotton-cloth, raw hides, wheat-bread, telephones?
  7. Explain what is meant by a fall in the value of money; an appreciation of gold; a depreciation of inconvertible paper; a stable standard of value.
  8. Wherein does the play of demand and supply, in determining the value of money, differ from its operation in determining the value of commodities in general? Wherein does cost of production determine the value of money and of commodities differently?
  9. What is the effect of general high wages on prices? on values? on profits? Why?
  10. “So far as rents, profits, wages, prices. are determined by competition, laws may be assigned for them. Assume competition to be their exclusive regulator, and principles of broad generality and scientific precision may be laid down, according to which they will be regulated.” Trace the historical origin of the conditions here assumed.
  11. What seems to you to be the value of economic history in relation to the study of economic theory?

Source: Harvard University Archives. Harvard University, Mid-year examinations, 1852-1943. Box 3, Volume: Examination Papers, Mid-Year, 1893-94.

1893-94.
ECONOMICS 1.
Final Examination.

[Arrange your answers strictly in the order of the questions.]

I.
[One question in this group may be omitted.]

  1. Explain the connection between the law of diminishing returns the pressure of population on subsistence; the tendency of profits to a minimum.
  2. What is the nature of the remuneration received by the holder of a government bond; the holder of a railway bond; the landlord of a building let for business purposes; the landlord of land let for agricultural purposes; a manufacturer carrying on business with borrowed capital; the holder of a patent receiving a royalty for its use?
  3. How does cost of production influence tire value of (1) silver bullion, (2) oats, (3) coffee, (4) bicycles?
  4. What seems to you to be the value of economic history in relation to the study of economic theory?

II.
[One question in this group may be omitted.]

  1. In 1851, very rich deposits of gold were found in Australia. What would you expect the result to be in Australia on wages, prices, imports and exports?
  2. Is the gain from international trade to be found in the import or in the exports? Why and how?
  3. It is said that when the quantity of money is increased, prices rise precisely in proportion to the increase. What exceptions or qualifications would you make to this statement?
  4. Is the exportation of specie from a country disadvantageous?

III.
[Answer all in this group.]

  1. What sorts of advantages, in regard to wages, do Trade-unions and Coöperative Societies offer to workingmen?
  2. “Deposits are currency.” What is meant?
  3. What is the most important objection to the use of inconvertible paper money? What illustrations of its force do you find in the experience of the United States since 1860?
  4. Compare the policy followed in times of panic by the Bank of England, the Reichsbank of Germany, and the National Banks of the United States.

Source: Harvard University Archives. Harvard University, Examination papers, 1853-2001. Box 2, Volume: Papers Set for Final Examinations in Philosophy, History, Government and Law, Economics, Fine Arts, and Music in Harvard College, June 1894, pp. 34-35.

______________________

Economics 2.
Economic Theory from Adam Smith
to the Present Time.
1893-94.

Enrollment

For Graduates and Undergraduates:—

[Economics] 2. Professor Taussig. — Economic Theory from Adam Smith to the present time. — Examination of selections from leading writers. 3 hours.

Total 43: 12 Graduates, 16 Seniors, 10 Juniors, 1 Sophomore, 4 Others.

Source:   Harvard University, Annual Reports of the President and Treasurer of Harvard College, 1893-94, p. 61.

1893-94
ECONOMICS 2.
Mid-Year Examination.

[Arrange your answers strictly in the order of the questions.
Write with deliberation, but answer all the questions.]

  1. “It is no doubt true that a portion of capital is always remuneratory and not auxiliary in its nature; that is, does not consist of instruments that make labour more efficient, but of finished products, destined for the consumption of labourers and others. This part of capital continually becomes real wages (as well as real profits, interest, and rent), being purchased by the labourer with the money wages he receives from time to time. But it does not seem to me therefore correct to regard the real wages as capital ‘advanced’ by the employer to the labourer. The transaction between the two is essentially a purchase, not a loan. The employer purchases the results of a week’s labour, which thereby becomes part of his capital, and may be conceived — if we omit for simplicity’s sake the medium of exchange — to give the labourer in return some of the finished products of his industry.”
    Consider whether and how remuneratory capital continually becomes real interest and rent, as well as real wages; and give your opinion as to the closing analysis of the relation between employers and laborers.
  2. Suppose (1) that profit-sharing were universally adopted; (2) that laborers habitually saved a very large part of their income, — and consider whether any modification must be made in the reasoning of those who would maintain a Wages-Fund doctrine.
  3. It has been said that while the capital of the employing class is the immediate source from which wages are paid, the ultimate and important source is the income of the consumers who buy the goods made by the laborers for the capitalists. Consider this doctrine.
  4. Compare critically the treatment by Walker, Sidgwick, and Ricardo, of the relation between the profits of the individual capitalist and the amount of capital owned by him.
  5. State carefully Ricardo’s criticism of Adam Smith’s doctrine on labor as the measure of value.
  6. Compare Adam Smith’s reasoning with Ricardo’s as to the manner in which the progress of society in wealth affects profits.
  7. “We have seen that in the early stages of society both the landlord’s and the labourer’s share of the value of the produce of the earth would be but small; and that it would increase in proportion to the progress of wealth and the difficulty of procuring food. We have known, too, that although the value of the labourer’s portion will be increased by the high value of food, his real share will be diminished; while that of the landlord will not only be raised in value, but will also be increased in quantity.”
    Explain the reasoning by which Ricardo reached the several conclusions here summarized, and give your opinion as to the soundness of the conclusions.

Source: Harvard University Archives. Prof. F. W. Taussig, Examination Papers in Economics 1882-1935 (Scrapbook). Also: Harvard University Archives. Harvard University, Mid-year examinations, 1852-1943. Box 3, Volume: Examination Papers, Mid-Year, 1893-94.

1893-94
ECONOMICS 2.
[Final Examination.]

  1. “Perhaps the most striking conflict of the Wages-Fund-theory with facts, is found in the periodical influctions and depressions of trade. After a commercial crisis, when the shock is over and the necessary liquidation has taken place, we generally find that there is a period during which there is a glut of capital, and yet wages are low. The abundance of capital is shown by the low rate of interest and the difficulty of obtaining remunerative investments.” — Nicholson, Political Economy
    How far is the theory in conflict with the facts here adduced?
  2. How is the significance of the doctrine of consumer’s rent affected by the fact that the money incomes of different purchasers vary widely?
  3. Explain Marshall’s doctrine as to the influence on wages of the standard of living among laborers; and consider how far it differs from Richard’s teaching as to the connection between wages and the price of food.
  4. Explain Marshall’s doctrine of the quasi-rent of labor; compare it with his conclusions as to the rent of business ability; and point out how far he finds in either case something analogous to economic rent as defined by the classic writers.
  5. “It is not true that the spinning of yarn in a factory, after allowance has been made for the wear-and-tear of the machinery, is the product of the labour of the operatives. It is the product of their labour (together with that of the employer and subordinate managers) and of the capital; and that capital itself is the product of labour and waiting; and therefore the spinning is the product of labour (of many kinds) and of waiting. If we admit that it is the product of labour alone, and not of labour and waiting, we can no doubt be compelled by inexorable logic to admit that there is no justification for interest, the reward of waiting.”
    How far would you accept this reasoning?
  6. “Barter, though earlier historically than buying and selling, is really a mere complex transaction, and the theory of it is rather curious than important.” — Marshall.
    “The attribute of normal or usual value implies systematic and continuous production.” — Cairnes.
    “Where commodities are made for sale, the sellers’ subjective valuations fall out altogether, and price is determined by the valuation of the last buyer.” — Böhm-Bawerk.
    Explain these statements, separately or in connection with each other.
  7. What does Böhm-Bawerk mean by the general subsistence market, or the total of advances for subsistence; and how far do the “advances” differ from the wages-fund of the classic economists?
  8. Explain Böhm-Bawerk’s views as to the connection between the prolongation of the period of production, and the increase in the productiveness of labor; and consider how far his conclusions as to interest would need to be modified, if those views were changed.
  9. Explain briefly, by definition or example, the sense in which Böhm-Bawerk uses the terms, —

social capital;
private capital;
subjective value;
marginal pairs;
technical superiority of present goods.

Source: Harvard University Archives. Prof. F. W. Taussig, Examination Papers in Economics 1882-1935 (Scrapbook). Also: Harvard University Archives. Harvard University, Examination papers, 1853-2001. Box 2, Volume: Papers Set for Final Examinations in Philosophy, History, Government and Law, Economics, Fine Arts, and Music in Harvard College, June 1894, pp. 35-36.

___________________________

Economics 3.
Principles of Sociology.
1893-94.

Enrollment.

[Economics] 3. Asst. Professor Cummings. — The Principles of Sociology. — Development of the Modern State, and of its Social Functions. 3 hours.

Total 47: 17 Gr., 19 Se., 5 Ju., 6 Others.

Source: Harvard University. Report of the President of Harvard College, 1893-94, p. 61.

 

1893-94.
ECONOMICS 3.
Mid-Year Examination

(Arrange your answers in the order of your questions. Omit two.)

  1. “In fact, the conception of society as an organism seems to admit of more easy application to just those very views about the State which Mr. Spencer most dislikes: and, though the conception or organism has its value in helping political thinking out of the confusions of individualism, if it be taken as a final key to all mysteries, it leads to new confusions of its own, for which it would be absurd to blame Mr. Spencer.” Explain and criticise.
  2. How does Spencer account for the diverse types of political organization; and what influences determine the order in which they arise? Illustrate.
  3. What evidence of political evolution is there in the sequence of the various forms of political organization in Greek, Roman, and Medieval society? Trace the steps.
  4. According to Burke, “Society is indeed a contract. … It is a partnership in all science; a partnership in all art; a partnership in every virtue and in all perfection. As the ends of such a partnership cannot be obtained in many generations, it becomes a partnership not only between those who are living, but between those who are living, those who are dead, and those who are to be born.” Explain. How does this differ from earlier conceptions of the social contract? From the conception of society as an organism?
  5. Upon what grounds does Spencer base his preference for the industrial rather than the militant type of society?
  6. According to Jevons, “the first step must be to rid our minds of the idea that there are any such things in social matters as abstract rights, absolute principles, indefeasible laws, inalterable rules, or anything whatever of an eternal and inflexible nature.” According to another view, “the state presupposes rights and the rights of individuals.” What is your own opinion? Why? Are there “Natural Rights”? Illustrate.
  7. “The State is after all the least of the powers that govern us.” How far is this true at different stages of social development?
  8. What is involved in the conception of Sovereignty? In whom is it rested? On what does it rest? For example, England and the United States.
  9. What is the bearing of Comte’s maxim, “Voir pour prevour,” upon the doctrine of social evolution?
  10. “The environment in our problem must, therefore, not only include psychical as well as physical factors, but the former are immeasurably the more important factors, and as civilization advances their relative importance steadily increases.”
  11. What do you mean by State Interference? By Individual liberty?

Source: Harvard University Archives. Harvard University, Mid-year examinations, 1852-1943. Box 3, Volume: Examination Papers, Mid-Year, 1893-94.

1893-94.
ECONOMICS 3.
Final Examination

[Questions are in all cases to be discussed with direct reference to facts and theories presented in this course. Arrange your answers in the order in which the questions stand. Take either the first question or six others.]

  1. Devote three hours to a discussion of “Social Evolution”;— expounding Mr. Kidd’s views, discussing his opinions and conclusions in the light of facts and theories presented in this course, and stating carefully your own reasons for agreeing or disagreeing.
  2. What, according to Mr. Kidd, are the necessary “Conditions of Human Progress”? Do you agree or disagree? Why?
  3. What are the points of resemblance and of difference between the “Scientific Socialism” of today and earlier forms of so-called socialistic propaganda which have appeared within this century?
  4. “Step by step the community has absorbed them, wholly or partially, and the area of private exploitation has been lessened. Parallel with this progressive nationalization or municipalization of industry, there has gone on, outside, the elimination of the purely personal element in business management.” Indicate briefly the character, extent and probable significance of “nationalization and municipalization” in the United States and in European Countries.
  5. What inferences may and what may not safely be drawn from American experience in municipal ownership or control of gas, of water, and of electric light plants? Discuss carefully the extent and character of the evidence.
  6. “According to them, the tribe or horde is the primary social unit of the human race, and the family only a secondary unit, developed in later times. Indeed, this assumption has been treated by many writers, not as a more or less probable hypothesis, but as a demonstrated truth. Yet the idea that a man’s children belong to the tribe, has no foundation in fact.” Indicate briefly the present state of this controversy. What significance do you attach to it?
  7. “The central fact with which we are confronted in our progressive societies is, therefore, that the interests of the social organism and those of the individuals comprising it at any time are actually antagonistic; they can never be reconciled, they are inherently and essentially irreconcilable.” State carefully the arguments for and against this position.
  8. “True Socialism of the German type must be recognized to be, ultimately, as individualistic and as anti-social as individualism in its advanced forms.” By what line of reasoning is this conclusion reached? State carefully your reasons for agreeing or disagreeing.

Source: Harvard University Archives. Harvard University, Examination papers, 1853-2001. Box 2, Volume: Papers Set for Final Examinations in Philosophy, History, Government and Law, Economics, Fine Arts, and Music in Harvard College, June 1894, pp. 36-37.

___________________________

Economics 5 (First Semester).
Railway Transportation.
1893-994.

Enrollment.

[Economics] 51. Professor Taussig. — Railway Transportation. — Lectures and written work. 3 hours. 1st half-year.

Total 39: 3 Gr., 24 Se., 9 Ju., 1 So., 2 Others.

Source: Harvard University. Report of the President of Harvard College, 1893-94, p. 61.

1893-94.
ECONOMICS 5.
Final [Mid-Year] Examination.

[Arrange your answers strictly in the order of the questions.]

  1. State what important general lessons are to be learned from the early experiments of Pennsylvania and Michigan in constructing and managing transportation routes.
  2. Why the change in the attitude of the public towards the Pacific railways after 1870? And what were some consequences of the change?
  3. What was the effect of the land-grant system on the welfare of the community, and on railway profits?
  4. “These conditions [leading to financial losses] may fairly enough be described as the Interstate Commerce Commission describes them, — parallel railroad construction and wars of rates. But when the Commission goes on to say that they cannot with any justice be claimed to have resulted from the act or from its administration, they make an unwarranted assertion.” What were the conditions here referred to (give dates)? And was the assertion unwarranted?
  5. Consider the probable results of the repeal of the section of the Interstate Commerce act which prohibits pooling.
  6. “High rates on some articles are not to be regarded as a tax which could be removed if low rates on others were abandoned.” Why not?
  7. “The enormous fixed capital and the consequent impossibility of retiring from the enterprise if it becomes unprofitable; the greater or less degree of monopoly; the wide gulf between railway managers and investors, sometimes leading to consequences of its own,” consider in what manner and extent these circumstances have affected railway rates in the United States.
  8. What do you believe to be the significance and importance of the following figures (for the United States in 1891):
Revenue per passenger mile 2.142 cents
Average cost of carrying a passenger one mile 1.910 cents
Revenue per ton mile 0.895 cents
Average cost of carrying a ton one mile 0.583 cents
Revenue per freight train mile $1.63
Average cost of running a freight train one mile $1.06
  1. Compare the course of railway policy in France, Prussia, and Italy, in 1880-85.
  2. Compare the principles which underlie the natural (car-space) system of freight rates and the zone system of passenger rates.

Source: Harvard University Archives. Harvard University, Examination papers, 1853-2001. Box 2, Volume: Papers Set for Final Examinations in Philosophy, History, Government and Law, Economics, Fine Arts, and Music in Harvard College, June 1894, pp. 37-38.

___________________________

Economics 6 (Second Semester)
History of Tariff Legislation
in the United States.
1893-94.

Enrollment.

[Economics] 62. Professor Taussig. — History of Tariff Legislation in the United States. 3 hours. 2d half-year.

Total 97: 11 Gr., 33 Se., 36 Ju., 2 So., 1 Fr., 14 Others.

Source: Harvard University. Report of the President of Harvard College, 1893-94, p. 62.

1893-94.
ECONOMICS 6.
Final Examination

[Arrange your answers strictly in the order of the questions.
Answer all the questions.]

  1. Is it to be inferred from Hamilton’s Report on Manufactures that if he were now living, he would not be an advocate of protection?
  2. What grounds are there for saying that the act of 1789 was a protective measure?
  3. State the important provisions of the act of 1816, and consider whether it differs in any essentials from the act of 1824.
  4. Was Clay right in affirming, or Webster in denying, that the protective system of 1824 was “American”?
  5. How would you ascertain what were the duties, in 1840, on (1) woollen goods, (2) cotton goods, (3) silk goods, (4) bar iron?
  6. Suppose the present specific duties on woollen manufactures to be removed; the ad valorem duties to remain unchanged; wool to be admitted free; and consider how far there would ensue a change in the effective protection given on finer woollen cloths, on cheaper woollen cloths, and on carpets.
  7. Mention briefly what were the duties on tea and coffee in the successive stages of tariff legislation from 1789 to 1890; noting the significance of the changes made from time to time.
  8. Why do the effects, in recent times, of the duties on flax and hemp, and on glassware, “reduce themselves in the last analysis to illustrations of the doctrine of comparative costs”?
  9. Wherein is there resemblance, wherein difference, between the general course of tariff history in the United States after the civil war, and in France after the Napoleonic wars?
  10. What would be the probable effects of the removal of the present duties on cotton goods?

Source: Harvard University Archives. Harvard University, Examination papers, 1853-2001. Box 2, Volume: Papers Set for Final Examinations in Philosophy, History, Government and Law, Economics, Fine Arts, and Music in Harvard College, June 1894, pp. 38-39

___________________________

Economics 8 (First Semester)
History of Financial Legislation
in the United States.
1893-94.

Enrollment.

[Economics] 81. Professor Dunbar. — History of Financial Legislation in the United States. 2 hours. 1st half-year.

Total 63: 9 Gr., 26 Se., 23 Ju., 1 So., 4 Others.

Source: Harvard University. Report of the President of Harvard College, 1893-94, p. 62.

1893-94.
ECONOMICS 8.
Mid-Year Examination.

Instead of answering the starred questions in this paper you may substitute, if you prefer, an essay on the subject marked A, printed at the close.

  1. *“It is sometimes said that Mr. Hamilton believed in a perpetual debt, and when one notices the form into which he threw the obligations of the United States, the only escape from this conclusion is to say that he was ignorant of the true meaning of the contracts which he created.” — [H. C. ADAMs, Public Debts, p. 161.]
    How far is the above remark confirmed by the provisions as to the payment of the debt funded by the Act of 1790?
  2. How far should you say that Gallatin, although an anti-Federalist, finally adopted Federalist measures or methods in financial matters?
  3. Give a general statement of the agreement between the banks and the Treasury for the resumption of specie payment in 1817, and show the way in which it was intended to operate.
  4. Inasmuch as Jackson’s general prepossessions were unfavorable to all banks, how are we to explain his resort to the plan of depositing Government funds in State banks after the removal of the deposits in 1833?
  5. *How serious a blow did Jackson really strike when he removed the deposits from the United States bank in 1833?
  6. What expedients were suggested for supplying the needs of the government in 1861-62 without resorting to the issue of legal-tender notes?
  7. *The “Gold Bill” of June 17, 1864, and its fate.
  8. What was the process by which the bonds issued during the war were refunded under the act of 1870 and when did the refunding take place?
  9. What signs of change in the policy of Congress as to the resumption of specie payments are to be found in the legislation between 1865 and 1876?
  10. State the provisions of the Resumption Act of 1875 as to the redemption of legal-tender notes, and show whether the act did or did not provide for the possible eventual disappearance of all the notes. What has made the amount of outstanding legal-tender notes stationary at $346,681,016?
  11. *A recent writer, discussing the question of a paper currency issued by government, says:—
    “In the United States there were twenty issues of treasury notes before the late war. Those issues were receivable in the revenues the government, and were always preferred to gold.”
    What criticism is to be made on this statement?
  12. *Describe the different kinds of paper currency now in use in the United States, stating as to each the cases in which it can be tendered for private debt, and those which it. can he received or paid out by the government.

*  *  *  *  *  *  *  *  *  *
A.

The change which has taken place since 1846 in the conditions affecting the Independent Treasury, and the justification of Secretary Carlisle’s statement, in the Finance Report for 1893, that “the laws have imposed upon the Treasury Department all the duties and responsibilities of a bank of issue, and to a certain extent the functions of bank of deposit.”

Source: Harvard University Archives. Harvard University, Mid-year examinations, 1852-1943. Box 3, Volume: Examination Papers, Mid-Year, 1893-94.

___________________________

Economics 9.
The Social and Economic Condition of Workingmen in the United States and in other countries.
1893-94.

 Enrollment.

[Economics] 9. Asst. Professor Cummings. — The Social and Economic Condition of Workingmen in the United States and in other countries. 3 hours.

Total 43: 7 Graduates, 16 Seniors, 11 Juniors, 3 Sophomores, 1 Freshman, 5 Other.

Source: Harvard University. Annual Report of the President of Harvard College, 1893-94, p. 61.

Mid-year Examination.
ECONOMICS 9.
1893-94.

(Arrange your answers in the order in which the questions stand. So far as possible illustrate your discussions by a comparison of the experience of different countries. Omit two questions.)

  1. “It becomes my duty, therefore, in undertaking to interpret the social movement of our own times, to disclose, first, those changes in industrial methods by which harmony in industries has been disturbed, and then to trace the influence of such changes into the structure of society.” State carefully what these changes have been; and trace their influence.
    [Henry C. Adams. “An Interpretation of the Social Movements of our Time”, International Journal of Ethics, Vol II, October, 1891), p. 33]
  2. Discuss the effect upon wages of machinery, — (a) as a substitute for labor (b) as auxiliary to labor; (c) as affecting division of labor; (d) as concentrating labor and capital; (e) as affecting the nobility[sic, “mobility”] of labor and capital.
  3. “In my opinion, combination among workingmen is a necessary step in the re-crystallization of industrial rights and duties.” State fully your reasons for agreeing or disagreeing with this opinion. What forms of combination do you include?
    [Henry C. Adams. “An Interpretation of the Social Movements of our Time”, International Journal of Ethics, Vol II, October, 1891), p. 45]
  4. “Trade-unions have been stronger in England than on the Continent, and in America….” In what respects stronger? Why? Contrast briefly the history and present tendencies of the trade-union movement in the United States, England, France, Germany, and Italy.
    [Alfred Marshall, Elements of Economics of Industry: being the First Volume of Elements of Economics (London: Macmillan, 1892), Book VI, Ch. XIII. §18, p. 404]
  5. “Trade-unions have been stronger in England than on the Continent, and in America; and wages have been higher in England than on the Continent, but lower than in America.” “Again, those occupations in which wages have risen most in England happen to be those in which there are no unions.” How far do such facts impeach the effectiveness of trade-unions as a means of raising wages and improving the condition of workingmen? What do you conceive to be the economic limits and the proper sphere of trade-union action?
    [Alfred Marshall, Elements of Economics of Industry: being the First Volume of Elements of Economics (London: Macmillan, 1892), Book VI, Ch. §18, pp. 404-405.]
  6. “We saw at the beginning that in comparatively recent years the difficulties of keeping up a purely offensive and defensive organization had brought many of the unions back nearer their old allies, the friendly societies, and emphasized the friendly benefits in proportion as the expenditure for trade disputes seemed less important.” Explain carefully this earlier and later relation of trade-unions and Friendly Societies in England.
    [Edward Cummings, The English Trades-Unions, Quarterly Journal of Economics, Vol. III (July, 1889), p. 432.]
  7. “This spirit of independent self-help has its advantages and its disadvantages. We have already had occasion to remark how slow in these Friendly Societies has been the progress of reform, and we must repeat that up to the present day it still exhibits defects.” Explain and illustrate the progress of the reform and the nature of existing defects. Does English self-help experience suggest the desirability or undesirability of imitating German methods of compulsory insurance?
  8. “Countless[sic, “Doubtless” in original] boards of arbitration and conciliation, the establishment of certain rules of procedure, agreements covering definite periods of time, may aid somewhat in averting causes of dispute or in adjusting disputes as they arise; but if we have these alone to look to, strife will be the rule rather than the exception.” Explain the various methods adopted and the results obtained. What have you to say of “compulsory arbitration?”
    [Francis A. Walker. “What Shall We Tell the Working Classes?” Scribner’s Magazine, Vol. 2, 1887.  Reprinted in Discussions in Economics and Statistics, edited by Davis R. Dewey. Vol. II315-316.]
  9. “The conclusion of the whole matter seems to be, that what is desirable is not so much to put a stop to sub-contracting as to put a stop to ‘sweating,’ whether the man who treats the workman in the oppressive manner which the word ‘sweating’ denotes be a sub-contractor, a piece-master, or a contractor.” Indicate briefly some of the principal forms of industrial remuneration, — giving the special merits and defects of each.
    [David F. Schloss. Methods of Industrial Remuneration (London: Williams and Norgate, 1892), p. 140.]
  10. “Now that I am on piece-work, I am making about double what I used to make when on day-work. I know I am doing wrong. I am taking away the work of another man.” State and criticize the theory involved in this view of production.
    [David F. Schloss. Methods of Industrial Remuneration (London: Williams and Norgate, 1892), p. 43-44.]

Source: Harvard University Archives. Harvard University, Mid-year examinations, 1852-1943. Box 3, Volume: Examination Papers, Mid-Year, 1893-94. Transcribed and posted earlier at Economics in the Rear-view Mirror.

Year-End Examination
ECONOMICS 9.
1893-94.

(Arrange your answers in the order in which the questions stand. So far as possible illustrate your discussions by a comparison of the experience of different countries. Take the first three questions and four others.)

  1. “As soon, however, as the factory system was established, the inequality of women and children in their struggle with employers attracted the attention of even the most careless observers; and, attention once drawn to this circumstance, it was not long before the inequality of adult men was also brought into prominence.” How far is this true (a) of England, (b) of the United States? Trace briefly the legislative consequences for children and for adults in the two countries.
    [Arnold Toynbee. Lectures on the Industrial Revolution of the 18th Century in England (The Humboldt Library of Popular Science Literature, Vol. 13. New York: Humboldt Publishing Co.), p. 17.]
  2. “It will be necessary, in the first place, to distinguish clearly between the failure of Industrial Coöperation and the failure of the coöperative method—a method, as we have seen, adopted, even partially, by only a very small fraction of Industrial Coöperation.” Explain carefully, discussing especially the evidence furnished by France and England.
  3. “These four concerns—the Maison Leclaire, the Godin Foundry, the Coöperative Paper Works of Angoulême and the Bon Marché—are virtually coöperative; certainly they secure to the employers and stockholders the substantial benefits of purely coöperative productive enterprises, while they are still, logically, profit-sharing establishments.” State your reasons for agreeing or disagreeing. Indicate briefly the characteristic features of each enterprise.
  4. “What inferences are we to draw from the foregoing statistics? Unmistakably this, that the higher daily wages in America do not mean a correspondingly enhanced labor cost to the manufacturer. But why so?” Discuss the character of available evidence in regard to the United States, Great Britain and the continent of Europe.
    [E. R. L. Gould. The Social Condition of Labor (Baltimore: Johns Hopkins Press, January 1893), pp. 41-2.]
  5. “The juxtaposition of figures portraying the social-economic status of workmen of different nationalities in the country of their birth and the land of their adoption furnishes lessons of even higher interest. From this we are able to learn the social effect of economic betterment.” Explain. How do the facts in question affect your attitude toward recent changes in the character and volume of our immigration?
    [E. R. L. Gould. The Social Condition of Labor (Baltimore: Johns Hopkins Press, January 1893), pp. 35-6.]
  6. “The Senate Finance Committee issued some time ago a comparative exhibit of prices and wages for fifty-two years, from which the conclusion is generally drawn that the condition of the wage earner is better to-day than it was thirty or forty years ago. A conclusion of this kind reveals the weakness of even the best statistics. No one can doubt that the work of the Finance Committee is work of high excellence, but for comparing the economic condition of workers it is of little value.” Do you agree or disagree? Why? Indicate briefly the character of the evidence.
  7. What are the principle organizations which may be said to represent the “Labor Movement” in the United States at the present time? How far are they helpful and how far hostile to one another?
  8. “In a preceding chapter I have said that as a moral force and as a system the factory system of industry is superior to the domestic system, which it supplanted.” State your reasons for agreeing or disagreeing.
    [Carroll D. Wright. Factory Legislation from Vol. II, Tenth Census of the United States, reprinted inFirst Annual Report of the Factory Inspectors of the State of New York (Albany, 1887), p. 41.]
  9. Contrast the English and the German policy in regard to Government Workingmen’s Insurance.
  10. “Gladly turning to more constructive work, I next consider some industrial changes and reforms which would tend to correct the present bias towards individualism.” What are they?
  11. Give an imaginary family budget for American, English and German operatives in one of the following industries, — coal, iron, steel, cotton, wool, glass, indicating roughly characteristic differences in such items as throw most light on the social condition of labor.

Source: Harvard University Archives. Harvard University, Examination papers, 1873-1915. Box 4, Volume: Examination Papers, 1893-95. pp. 39-41. Transcribed and posted earlier at Economics in the Rear-view Mirror.

______________________

Economics 10.
The Elements of Economic History from the Middle Ages to Modern Times.
1893-94.

[Economics] 10. Professor Ashley. — The Elements of Economic History from the Middle Ages to Modern Times. 3 hours.

Total 51: 6 Gr., 17 Se., 20 Ju., 4 So., 1 Fr., 3 Others.

Source: Harvard University. Annual Report of the President of Harvard College, 1893-94, p. 61.

1893-94.
ECONOMICS 10.
Mid-Year Examination.

 

  1. A modern writer has insisted upon the difference between the point of view of economic history and the point of view of constitutional history. Consider this in relation to the growth of mediaeval towns.
  2. Distinguish briefly between the various processes known as “Enclosure,” and explain their relation to the open-field husbandry.
  3. What light does the history of the English woollen industry throw upon the question as to the relation between the gild and the domestic workshop?
  4. “Only one who is unacquainted with social conditions under Henry VIll. and Edward VI. can maintain that the Reformation was not responsible for English pauperism.” Discuss this.

Source: Harvard University Archives. Harvard University, Mid-year examinations, 1852-1943. Box 3, Volume: Examination Papers, Mid-Year, 1893-94.

1893-94.
ECONOMICS 10.
Final Examination.

[Candidates are requested to answer only six questions, of which the first should be one.]

  1. Translate and comment upon:
    1. Omnes isti sochemanni habent viii carrucas, et arant iii vicibus per annum. Et quisquis eorum metit in Augusto de blado domini dimidiam acram et ii vicibus in Augusto precationem.
    2. Sciatis me concessisse … civibus meis in Oxenforde omnes libertates et consuetudines et leges et quietantias quas habuerunt tempore regis Henrici avi mei, nominatim gildam suam mercatoriam cum omnibus libertatibus et consuetudinibus in terris et in silvis pasturis et aliis pertinentiis, ita quod aliquis qui non sit de gildhalls aliquam mercaturam non faciet in civitate vel suburbiis.
  2. Give some account of the changes in trade-routes during the sixteenth century.
  3. Describe the organization of industry in the middle of the reign of Elizabeth.
  4. Compare the Enclosures of the eighteenth century with those of the sixteenth.
  5. What was the condition of the mercantile marine of New England in the eighteenth century? What connection was there between this condition and the Navigation Acts?
  6. Institute a comparison between the reforms of Stein and Hardenberg and recent agrarian legislation in Ireland, or any other country with which you are familiar.
  7. What light is cast upon the teaching of (1) Adam Smith, (2) Malthus, (3) Ricardo, by contemporary economic conditions.
  8. Estimate the importance of Arthur Young in the economic history of England.
  9. What seem to you the most characteristic features of the economic development of the United States during the present century as contrasted with England.

Source: Harvard University Archives. Harvard University, Examination papers, 1853-2001. Box 2, Volume: Papers Set for Final Examinations in Philosophy, History, Government and Law, Economics, Fine Arts, and Music in Harvard College, June 1894, pp. 42-43.

___________________________

Economics 12 (First Semester).
Banking and the History
of the leading Banking Systems
1893-94.

 Enrollment.

[Economics] 121. Professor Dunbar. — Banking and the History of the leading Banking Systems. 3 hours. 1st half-year.

Total 50: 10 Gr., 24 Se., 15 Ju., 1 Other.

Source: Harvard University. Report of the President of Harvard College, 1893-94, p. 62.

1893-94.
ECONOMICS 12[1].
Mid-Year Examination

  1. Which system of banks appears to present the greatest advantages, — (a) one with a powerful central bank as in England and Germany: (b) an aristocracy of strong banks as in Scotland; or (c) a democracy of banks as in this country?
  2. In any period of financial pressure, would the Bank of England he under any obligation, legal or moral, to act for the relief of the public, if such action involved risk or loss to its stockholders? What would be the source of such obligation, if any exists?
  3. The German bank act requires every bank to hold cash, (a) for all notes issued by it above its limit of uncovered issue: (b) and amounting to at least one third of all the notes issued Why is it that notes of other banks can be reckoned as cash in one of these cases, but not in the other?
  4. What is to be said as to the proposition frequently maintained. that “note issue is in reality a function of the State as much as coinage, and should not be delegated to corporations or to private hands?”
  5. If we hold that all note issues need to be kept under national control, in order to secure uniformity of value, what ground is there for denying that all deposit banking needs the same control for the same reason?
  6. Supposing the securities required for deposit under the national banking system to be abundant and fairly attractive as investments, — would that system afford an elastic currency?
  7. To the plan of securing notes by a safety fund (as practiced formerly in New York and now in Canada), it has been objected that it would be unjust to require well-managed banks to pay for losses incurred by weak or imprudent ones, and that a premium would be offered for bad management. How much weight is there in this objection?
  8. To the plan of making the notes of a bank a first lien on its assets it has been objected,—
    “It deprives the bank of the fund which is the basis of its credit in asking for deposits Without the deposit the banks cannot do a profitable business. It is difficult to believe that, the capital being subjected to a first lien for the amount of the notes, and there being always the possibility of an over-issue of such notes, the credit of the bank in its discount and deposit business would not be impaired. is calling upon the capital to do a double work when it is already loaded with the single task of inspiring confidence in the people who have to make deposits.”
    What is the answer to this objection?
  9. Discuss the following extract from the Commercial and Financial Chronicle of May 14th, 1892:—
    “Every prerogative and attribute even of our bank notes, and still more of our silver certificates, tends to draw them away from the interior, even when the issuer is resident in a Southern or Western State, and lodge them in an Eastern city. [The semi legal-tender quality of the national bank circulation and its redemption at the Treasury help to make its movements unnatural, artificial, and impart to it a roaming character helping to force it away from the issuer, away from the country districts where it is needed, and consequently to induce its accumulation when out of active commercial employment in the great financial centres, and while there to foster and become more or less fixed in speculative ventures — that is unresponsive to commercial influences when needed for commercial work?”

Source: Harvard University Archives. Harvard University, Mid-year examinations, 1852-1943. Box 3, Volume: Examination Papers, Mid-Year, 1893-94.

___________________________

Economics 12 (Second Semester).
International Payments and the Flow of the Precious Metals.
1893-94.

Enrollment.

[Economics] 122. Professor Dunbar.—International Payments and the Flow of the Precious Metals. 3 hours. 2d half-year.

Total 38: 12 Graduates, 18 Seniors, 7 Juniors, 1 Other.

Source: Harvard University. Report of the President of Harvard College, 1893-94, p. 62.

1893-94
ECONOMICS 122.
Final Examination.

  1. Mr. Goschen says that while a gold currency existed on both sides of the Atlantic the actual par of exchange between New York and London was about 109. What is the explanation of this method of stating the point of equilibrium?
  2. Is Clare justified in making the general statement that “the gold-points mark the highest level to which an exchange may rise, and the lowest to which it may fall?”
  3. What effect would the current rate of interest (as e.g. in a tight money market, either in the drawing or in the accepting country,) have on the rates for sixty-day bills as compared with cash bills?
  4. Clare makes the remark that “as the rate of exchange between two countries…must be fixed by the one who draws and negotiates the bill, it follows that the exchanges between England and most other countries are controlled from the other side, and that we in London have scarcely part or say in the matter.” Is the rate then a matter of indifference to those in London?
  5. Why is it that in certain trades bills are drawn chiefly, or even exclusively, in one direction, as e.g. by New York on London and not vice versa; and how is this practice made to answer the purpose of settling payments, which have to be made in one direction as well as the other?
  6. Mr. Goschen says that the primary cause which makes England the great banking centre of the world is “the stupendous and never-ceasing exports of England, which have for their effect that every country I the world, being in constant receipt of English manufactures, is under the necessity of making remittances to pay for them, either in bullion, in produce, or in bills.”
    Compare this statement with the fact that for ten years past the imports of merchandise into England have averaged about £400,000,000 annually, and the exports from England have averaged a little under £300,000,000.
  7. Suppose the exportation of specie from the United States to be prohibited (or, as has sometimes been suggested, to be slightly hindered,) what would be the effect on rates of exchange, and on prices of goods, either domestic or foreign? Would the country be a loser or not? [See Ricardo (McCulloch’s ed.) p. 139.]
  8. State Mr. Cairnes’s general doctrine as to the movement of prices which determines the normal flow of new supplies of gold from one country to another in the process of distribution over the commercial world.
  9. Cairnes argues that, as the effect of the cheapening of gold, “each country will endure a loss;” but that in particular cases “the primary loss may…be compensated, or even converted into a positive gain.” State and discuss the reasoning on which this proposition rests.
  10. Say, in his Report on the Indemnity, says:—
    La France a, en réalité, (1) fait passer à l’étranger le plus de capitaux possible, en prenant tous les changes qu’elle pouvait acquérir sur quelque pays que ce fût, et (2) a ensuite dirigé sur l’Allemagne tout ce qu’elle avait approvisionné ailleurs.

    1. What reason was there why France should prefer the course described in (1) rather than a direct transfer to Germany?
    2. What movements of trade or capital, of any sort, made the course described in (1) possible or easy?
    3. What movements of the same nature made (2) possible, or enable Germany to absorb the capital thus turned towards her?

*  *  *  *  *  *  *

  1. On either of the following topics, give an orderly and concise statement, as complete as you can make it in thirty minutes:—
    1. Sidgwick’s criticisms on Mill’s doctrine of international trade and their validity.
    2. The supply and distribution of the new gold from the United States and Australia, 1858-70.
    3. The action of the new gold in the banking countries.
    4. The absorption of new gold by the currency of France and the foreign trade of that country.
    5. The reasons for the varying ability of India to absorb silver?

Source:  Harvard University Archives. Final examinations, 1853-2001. Box 2, Papers set for Final Examinations in Philosophy, History, Government and Law, Economics, Fine Arts, and Music in Harvard College, June 1894, pp. 44-46. Transcribed and posted earlier in Economics in the Rear-view Mirror.

____________________

1893-94
Enrollment for Economics 13.
The Development of Land Tenures and of Agrarian Conditions in Europe.

Enrollment.

[Economics] 13. Professor Ashley. – The Development of Land Tenures and of Agrarian Conditions in Europe. 1 hour.

Total 2: 1 Graduate, 1 Senior.

Source: Harvard University. Report of the President of Harvard College, 1893-94, p. 61.

Note: No printed final examination in the collection of Harvard semester examinations.

____________________

Economics 14.
Ideal Social Reconstructions
from Plato to the Present.
1893-94.

Enrollment.

[Economics] 14. Asst. Professor Cummings. — Ideal Social Reconstructions, from Plato’s Republic to the present time. 1 hour.

Total 22: 7 Graduates, 8 Seniors, 5 Juniors, 2 Sophomores.

Source: Harvard University. Report of the President of Harvard College, 1893-94, p. 61.

ECONOMICS 14.
Mid-year examination, 1893-94.

(Arrange your answers in the order of the questions. Omit one.)

  1. What is a Utopia? and what significance do you attached to the recurrence of such literature at certain historical ethics?
  2. “For judging of the importance of any thinker in the history of Economics, no matter is more important to us than the view he takes of the laboring population.” Judge Plato, More and Bacon by this standard.
  3. “Moreover, it is hardly too much to say that Plato never got to the point of having a theory of the State at all.” In the Republic “man is treated as a micropolis, and the city is the citizen writ large.” Explain and criticize.
  4. “In More’s Utopia we have a revival of the Platonic Republic with additions which make the scheme entirely modern.… The economical element in the social body receives for the first time its proper rank as of the highest moment for public welfare.” Explain. To what extent have the ideals of Utopia been realized?
  5. “Then we may say that democracy, like oligarchy, is destroyed by its insatiable craving for the object which defines to be supremely good?” What, according to the Republic are the peculiar merits and defects of the several forms of political organization? and how are these forms related in point of origin and sequence?
  6. “Sir Thomas More has been called the father of Modern Communism.” How does he compare in this respect with Plato? How far do you trace the influence of historical conditions in each case?
  7. “But in your case, it is we that have begotten you for the State as well as for yourselves, to be like leaders and kings of the hive,– better and more perfectly trained than the rest, and more capable of playing a part in both modes of life.” Criticise the method and purpose of the educational system of the Republic. How far does Plato’s argument as to the duty of public service apply to the educated man to-day?
  8. “The religious ferment produced by the Reformation movement had begun to show signs of abatement, when another movement closely connected with it made its appearance almost at the same time in England and Italy, namely, the rise of a new philosophy.” How was this new philosophy embodied in the social ideals of Bacon and of Campanella? and what is the distinguishing characteristic of it?
  9. What essential contrast between pagan and Christian ideals have you found in schemes for social regeneration?
  10. Is there any recognition of “Social Evolution” in the Utopian philosophies thus far considered?
  11. What in a word, do you regard as the chief defect of the social reconstruction suggested in turn by Plato, Lycurgus, More, Bacon and Campanella? To what main problems suggested by them have we still to seek an answer?

Source: Harvard University Archives. Harvard University, Mid-year examinations, 1852-1943. Box 3, Volume: Examination Papers, Mid-Year, 1893-94. Previously transcribed and posted in Economics in the Rear-view Mirror.

 

ECONOMICS 14.
Final examination, 1893-94.

(Arrange your answers in the order of the questions.)

  1. [“]The essential unity and continuity of the vital process which has been in progress in our civilization from the beginning is almost lost sight of. Many of the writers on social subjects at the present day are like the old school of geologists: they seem to think that progress has consisted of a series of cataclysms.” How far is this criticism true? Is the characteristic in question more or less conspicuous in earlier writers?
  2. “At the outset underneath all socialist ideals yawns the problem of population…. Under the Utopias of Socialism, one of two things must happen. Either this increase must be restricted or not. If it be not restricted, and selection is allowed to continue, then the whole foundations of such a fabric as Mr. Bellamy has constructed are bodily removed.” State carefully your reasons for agreeing or disagreeing. In which of the schemes for social reconstruction, ancient or modern, do you find any adequate recognition of the part which selection plays in progress?
  3. “If it is possible for the community to provide the capital for production without thereby doing injury to either the principle of perfect individual freedom or to that of justice, if interest can be dispensed with without introducing communistic control in its stead, then there no longer stands any positive obstacle in the way of the free social order.” Discuss the provisions by which Hertzka hopes to guaranteed this “perfect individual freedom.” Contrast him with Bellamy in this respect.
  4. “I perceive that capitalism stops the growth of wealth, not – as Marx has it – by stimulating ‘production for the market,’ but by preventing the consumption of the surplus produce; and that interest, though not unjust, will nevertheless in a condition of economic justice becomes superfluous and objectless.” Explain Hertzka’s reasoning and criticise the economic theory involved.”
  5. What is the gist of “News from Nowhere”?
  6. The condition which the social mind has reached may be tentatively described as one of realization, more or less unconscious, that religion has a definite function to perform in society, and that it is a factor of some kind in the social evolution which is in progress.” How far have you found a recognition of this factor in theories of social reconstruction?

Source: Harvard University Archives. Final Examinations, 1853-2001. (HUC 7000.28). Box 2, Papers Set for Final Examinations in Philosophy, History, Government and Law, Economics, Fine Arts, and Music in Harvard College, June 1894.

Also: Harvard University Archives. Harvard University, Examination papers, 1853-2001. Box 2, Volume: Papers Set for Final Examinations in Philosophy, History, Government and Law, Economics, Fine Arts, and Music in Harvard College, June 1894, pp. 46-47. Previously transcribed and posted in Economics in the Rear-view Mirror.

 

Source: Left-to-right: Dunbar, Taussig, Ashley. From University and their Sons. History, Influence and Characteristics of American Universities with Biographical Sketches and Portraits of Alumni and Recipients of Honorary Degrees. Editor-in-chief, General Joshua L. Chamberlain, LL.D. Vol II (1899), pp. 159 [Dunbar], 595 [Ashley].   Vol. III (1899), p. 99 [Taussig]

Categories
Chicago Funny Business

Chicago. Economics Christmas Skit Material, 1969

While no date is given for the following two pages, we can be confident that the material was prepared and one presumes performed at the Chicago Economics Department Christmas Party of 1969. Photos from the December 1970 Christmas party have been posted by Robert J. Gordon–they do not correspond to the texts below.

The events of campus unrest at Columbia, Cornell, Harvard and San Francisco State referred to all took place 1968-69, so the earliest possible date for this skit would have been in December 1969.

I have added the “true” lyrics to the chosen tunes as well as links to videos with the corresponding melodies for readers who wish to try their luck in the privacy of their own offices. Replication probably requires a cocktail or two to establish the appropriate a-critical mood. 

Your sober scribe was not particularly amused. OK, maybe the lighting, costuming, and orchestral arrangements were fantastic–hard to know. I pity though the poor future historians of present economics who will have to deal with audio and video evidence and not just the written record. 

________________________

SONGS FOR SKIT

University of Chicago
Economics Department
Skit Song Lyrics

“The Merry Minuet
(They’re rioting in Africa…)

https://youtu.be/L8-BI89mb9A

They’re rioting at C’lumbia

La La La La La La La

They’re shooting up Cornell

La La La La La

They’re plowin’ up ole Harvard Yard

La La La La La La La

And Hiyakowa’s catching hell.

La La La La La

Academia is festering with strife and discord

The faculty hate students cause they’re paranoid

But we can be certain and brimming with cheer

That none of this nonsense will ever happen here.

They’re rioting in Africa
They’re starving in Spain
There’s hurricanes in Florida
And Texas needs rain
The whole world is festering with unhappy souls
The French hate the Germans,
the Germans hate the Poles
Italians hate Yugoslavs,
South Africans hate the Dutch
And I don’t like anybody very much
But we can be thankful and tranquil and proud
That Man’s been endowed with the mushroom shaped cloud
And we know for certain that some lovely day
Some one will set the spark off and we will all be blown away
They’re rioting in Africa
There’s strife in Iran
What nature doesn’t do to us
Will be done by our fellow man!

 

University of Chicago
Economics Department
Skit Song Lyrics

Santa Claus is Coming to Town
https://youtu.be/HSmsq2iq4bQ
You’d better watch out
You’d better not strike
You’d better not riot
I’m (or We’re) telling you why
The National Guard is coming to town.
They know what you’ve been smoking
They know when you’ve been bad
They know when you’ve been sitting-in
So get out…do you understand!!
They’re making a list
And checking it twice
They’re going to find out
Whose [sic] Commie or nice
The National Guard is coming to town.
Oh! You better watch out
You better not cry
You better not pout
I’m telling you why
Santa Claus is coming to town
He sees you when you’re sleeping
He knows when you’re awake
He knows if you’ve been bad or good
So be good for goodness sake!
He’s making a list
Checking it twice
Gonna find out
Who’s naughty or nice
Santa Claus is coming to town

 

 

University of Chicago
Economics Department
Skit Song Lyrics

On Top of Old Smokey
https://youtu.be/P51eCjKN2Kw
On top of a mountain
In central Vermont
Resides Milton Friedman
Of wisdom the fount.
The scene is idyllic
On that mountain peak
But here in Chicago
The outlook is bleak.
Since Telser to Belgium
Has decided to roam,
Just Zecher and Gorden [sic]
Are left here at home.
No thesis prospectus
Are we able to give
Faculty all neglect us
As their prerogative.
Heed our ultimatum
Before it’s too late
Move the MONEY workshop
To the Green Mountain State.
On top of old smokey
all covered with snow
I lost my true lover
for courting too slow
For courting’s a pleasure
and parting’s a grief
And a false hearted lover
is worse than a thief
For a thief will just rob you
and take all you save
But a false hearted lover
will lead you to the grave
And the grave will decay you
and turn you to dust
Not one girl in a hundred
a poor boy can trust
They’ll hug you and kiss you
and tell you more lies
Than cross lines on a railroad
or stars in the skies
So come all your maidens
and listen to me
Never place your affections
on a green willow tree
For the leaves they will wither
and the roots they will die
You’ll all be forsaken
and never know why.

 

 

University of Chicago
Economics Department
Skit Song Lyrics

Mickey Mouse Club Song
https://youtu.be/x4C_lUy58Rw

Who’s the leader of the club
That’s made for you and me
M-i-l-t-o-n Da Da Da Da De[e]
Uncle Miltie,
Uncle Miltie
Forever let us sing his praises high
[…high, high, high]
He’s the man with just one theory
When others must use two
M-i-l-t-o-n Da Da Da Da Do[o]
Milt the Stilt (Paul the Small)
Milt the Stilt (Paul the Small)
In our hearts we know which one is  right […] [right, right, right]
Velocity is constant
The Phillips curve’s a fraud
M-i-l-t-o-n Da Da Da Da Da[w]
Money matters,
money matters
As long as prices
do not rise too fast.
What’s the purpose of the club
That’s made for you and I
U of C Ph.D. M-O-N-E-Y
Permanent Income,
Permanent income
It makes it all worthwhile, or so they[…]
[…]say. [say, say, say]
Rules and not discretion
And let me tell you why
M-I-L-T-O-N  M-O-N-E-Y
Who’s the leader of the club
That’s made for you and me
M-I-C-K-E-Y M-O-U-S-E
Hey! there, Hi! there, Ho! there
You’re as welcome as can be
M-I-C-K-E-Y M-O-U-S-E
Mickey Mouse! (Donald Duck)
Mickey Mouse! (Donald Duck)
Forever let us hold our banner
High! High! High! High!
Come along and sing the song
And join the jamboree!
M-I-C-K-E-Y M-O-U-S-E
Mickey Mouse club
Mickey Mouse club
We’ll have fun
We’ll meet new faces
We’ll do things and
We’ll go places
We’re marching all around the world
Who’s the leader of the club
That’s made for you and me
M-I-C-K-E-Y M-O-U-S-E
Hey! there, Hi! there, Ho! there
You’re as welcome as can be
M-I-C-K-E-Y M-O-U-S-E
Mickey Mouse! (Donald Duck)
Mickey Mouse! (Donald Duck)
Forever let us hold our banner
High! High! High! High!
Come along and sing a song
And join the jamboree!
M-I-C-K-E-Y M-O-U-S-E(yay Mickey)
(yay Mickey)
(yay Mickey Mouse Club!)

 

 

University of Chicago
Economics Department
Skit Song Lyric

 

O Tannenbaum (O Christmas Tree)

https://youtu.be/27JleM39TPY

Now that we’ve lost our faculties
To real world positions
We can observe to ascertain
What were their life ambitions
Lester Telser for his amusement
Investigated advertisement
So now we find him having fun
On the avenue called Madison.
Those who had taught development
Have left to form a settlement
With Harberger as President
An economist in residence
With [Larry] Sjastaad in an advisory task
They’re sure to find their golden path
And on their farms up with the sun
Are Teddy Schultz and Gale Johnson.
Bob Fogel has aspired to be
The president of the Santa Fee
Gregg Lewis we all should know
Leads the AFL and CIO
And Friedman’s gone up to Ely
To found his university
Big Harry with his knife so free
Now runs a toothpick factory.

[Handwritten addition:]

Uzawa + Mundell have gone to instigate at the Sorbonne
And [Erwin] Diewert is a lumberjack
Up near the straits of Mackinac

Geo. T who’s of urban fame [George S. Tolley]
Has taken over Lindsay’s game [NYC mayor]
And since there is no more faculty
We’ve all enrolled at MIT.

O Christmas Tree, O Christmas tree,
How lovely are your branches!
O Christmas Tree, O Christmas tree,
How lovely are your branches!
Not only green in summer’s heat,
But also winter’s snow and sleet.
O Christmas tree, O Christmas tree,
How lovely are your branches!
O Christmas Tree, O Christmas tree,
Of all the trees most lovely;
O Christmas Tree, O Christmas tree,
Of all the trees most lovely.
Each year you bring to us delight
With brightly shining Christmas light!
O Christmas Tree, O Christmas tree,
Of all the trees most lovely.
O Christmas Tree, O Christmas tree,
We learn from all your beauty;
O Christmas Tree, O Christmas tree,
We learn from all your beauty. 

 

Your bright green leaves with festive cheer,
Give hope and strength throughout the year.
O Christmas Tree, O Christmas tree,
We learn from all your beauty.

 

 

 

 

 

 

 

 

 

 

 

 

 

 

Some dialogue:

Opening scene, faculty seated around a table, one member is reading a newspaper:

One faculty member: (reading newspaper, shakes head) The students are revolting!

(All concur)

Another member: But thank God—ah I mean Milton—that we’re at Chicago. Our students are well behaved, well ordered, normal, continuous and homothetic.

Another: (questioning) But how do you know about their sex lives?

(Pause for a few seconds, for all the uproarious laughter, then break into song—“They’re rioting at Columbia….” [See above].)

(After song, and during, students enter, their spokesman present list of demands to Stigler).

Student spokesman: We’ve come to present our nonnegotiable demand schedule for reform in the department.

(All faculty in shock and dismay)

We have decided to bring the free market economy into the university. Therefore:

(1) We demand that prelim grades be bought and sold freely—thereby bringing greater efficiency into the production of economists.

(2) We demand the immediate return of all industrial organization exams from the public enterprise post office.

And (3) We demand the removal of all artificial floors and ceilings in the Department.

Stigler: (unrolls list of demands and exclaims) Heck—we’re saved. Your demand schedule is upward sloping (a pause)

(turns sheet of paper to audience)

And therefore nonexistent.

(All faculty sigh in relief)

 

Source: Harvard University Archives, Papers of Zvi Griliches, Box 129, Folder “Faculty skits, ca. 1960s”.

 

Categories
Chicago Exam Questions Suggested Reading Syllabus

Chicago. Syllabus and Final Exam, International Monetary Economics. Metzler, 1971

The two items below (syllabus and final exam) were incorrectly filed in Lloyd A. Metzler’s papers at Duke. I accidentally stumbled upon both today and thought that rather than trusting my memory of the locations of the syllabus and final exam for Economics 370 in 1971, I’d just transcribe and post the two artifacts today. According to the biographical note below, Lloyd Metzler retired from the University of Chicago in 1971 so this must have indeed been the last time that he taught international monetary economics at Chicago.

__________________________

Biographical Note

Lloyd Appleton Metzler was born on April 3, 1913 in Lost Springs, Kansas. He attended the University of Kansas, where he studied economics under John Ise and earned a Bachelor’s degree in 1935 and an MBA in 1938. Metzler then entered Harvard University. He served as an instructor and tutor at Harvard and completed a Ph.D. in economics in 1942. His dissertation, “Interregional Income Generation,” earned him the Wells Prize. That same year, Metzler was the recipient of a Guggenheim fellowship.

From Harvard, Metzler went on to Washington, D.C., where worked for the Office of Strategic Services and several economic policy and planning commissions between 1943 and 1946. Metzler joined the research staff of the Board of Governors of the Federal Reserve System in 1944. In 1946 he returned to academia when he accepted a teaching position at Yale University. He soon left Yale for the University of Chicago in 1947, where he remained for the rest of his career.

Dr. Metzler survived surgery for a brain tumor in 1952, and with the help of his wife Edith, managed to continue teaching and writing for the next twenty years. He served as Editor of the Journal of Political Economy from 1966 until his retirement in 1971. Metzler made numerous contributions to business cycle literature, macro-monetary theory, tariff theory, mathematical economics, and the field of international trade. The Metzler paradox, Laursen-Metzler effect, and Metzler matrix, all bear his name. He died on October 26, 1980.

Source: University of Chicago Library. Guide to the Lloyd A. Metzler Papers 1941-1948.

__________________________

For Intertemporal Comparison

Syllabus and readings for Economics 370 in 1950.

Exam for Economics 370 in 1953.

Syllabus, readings and final exam for Economics 370 from Winter Quarter 1967.

__________________________

ECONOMICS 370

Monetary Aspects of International Trade
Major Topics and Reading List
Winter, 1971

  1. Mechanism of the Foreign Exchange Market
    1. Alan R. Holmes and Francis Schott, The New York Foreign Exchange Market. New York: The Federal Reserve Bank of New York, 1965, Chapters 1-6.
    2. Frank A. Southard, Jr., Foreign Exchange Practice and Policy. New York: The McGraw-Hill Book Company, 1940.
    3. Norman Crump, The ABC of the Foreign Exchange. London: MacMillan and Company, Ltd., 1951.
    4. James E. Meade, The Theory of International Economic Policy: Vol. I. The Balance of Payments. London: Oxford University Press, 1951, Chapter 1.
  2. The Quantity of Money, the Rate of Interest, and the Price Level
    1. James Tobin, “The Monetary Policy and the Management of the Public Debt: The Patman Inquiry,” Review of Economics and Statistics, Vol. XXXV, No. 2, May 1953, pp. 118-27.
    2. Subcommittee on General Credit Control and Debt Management of the Joint Committee on the Economic Report, Hearings on the Question, What Should our Monetary and Debt Management Policy Be? 82nd Congress of the United States 1952, pp. 688-711, 691-98. (These pages include the testimony of Milton Friedman and Paul Samuelson).
    3. Robert V. Roosa, “Interest Rates and the Central Bank,” In Money, Trade, and Economic Growth, in honor of John Henry Williams, New York: The Macmillan Company, 1951.
    4. Lloyd A. Metzler, “Wealth, Saving, and the Rate of Interest,” Journal of Political Economy, Vol. LIX, No. 2, April 1951, pp. 93-116.
    5. Robert A. Mundell, “The Public Debt, Corporate Income Taxes, and the Rate of Interest,” Journal of Political Economy, Vol. LXVIII, No. 6, December 1960, pp. 622-26.
    6. George Horwich, “Real Assets and the Theory of Interest,” Journal of Political Economy, Vol. LXX, No.2, April 1962, pp. 158-69.
    7. Don Patinkin, Money, Interest, and Prices, 1st edition, Evanston: Row, Peterson and Company, 1956, Part II.
  3. The Role of Money in International Adjustment: Full Employment and Under-Employment
    1. J. M. Keynes, Treatise on Money: Vol. 1. The Pure Theory of Money. London: Macmillan and Company, 1935, Chapter 21.
    2. Lloyd A. Metzler, “The Theory of International Trade,” from A Survey of Contemporary Economics, Howard S. Ellis, Editor, Homewood, Ill.: R.D. Irwin, Inc., 1948.
    3. Lloyd A. Metzler, “The Process of International Adjustment Under Conditions of Full Employment: A Keynesian View,” first delivered before the Econometric Society, 1960, republished in AEA, Readings in International Economics, Vol. XI, Richard Caves and Harry Johnson, editors, Homewood, Ill.: R. D. Irwin, Inc., 1968, Chapter 28.
  4. Free-Market Exchange Rates
    1. A. J. Brown, “The Foreign Exchanges,” in Oxford Studies in the Price Mechanism, T. Wilson and P.W.S. Andrews, editors, Oxford at the Clarendon Press, 1951, Part II.
    2. Sidney Alexander, “Effects of a Devaluation on a Trade Balance,” International Monetary Fund Staff Papers, Vol. Il, No. 2, April 1952.
    3. Milton Friedman, “The Case for Flexible Exchange Rates,” in Essays in Positive Economics, Chicago: The University of Chicago Press, 1953, pp. 157-203.
    4. Joan Robinson, “The Foreign Exchanges,” in Essays in the Theory of Employment. Oxford: Basil Blackwell, 1947, Part III.
    5. Lloyd A. Metzler, “Exchange Rates and the International Monetary Fund,” in International Monetary Policies, Postwar Economic Studies, No. 7, Washington, D.C.: The Board of Governors of the Federal Reserve System, September 1947.
    6. Rudolph R. Rhomberg. “A Model of the Canadian Economy under Fixed and Fluctuating Exchange Rates,” Journal of Political Economy, Vol. LXXII, No. 1, February 1964, pp. 1-31.
  5. Forward Exchange Rates
    1. Paul Einzig, The Theory of Forward Exchange. London: Macmillan and Company, Ltd., 1937.
    2. Paul Einzig, A Dynamic Theory of Forward Exchang. London: Macmillan and Company, New York: St. Martin’s Press, 1961.
    3. Alan R. Holmes and Francis Schott, The New York Foreign Exchange Market. New York: The Federal Reserve Bank of New York, 1965, Chapters 7-8.
    4. Paul Einzig, “Some Recent Development in Official Forward Exchange Operations,” Economic Journal, Vol. LXXIII, No. 290, June 1963, pp. 241-53.
    5. Paul Einzig, “Some Recent Changes in Forward Exchange Practices,” Economic Journal, Vol. LXX, No. 279, September 1960, pp. 485-95.
  6. The Balance of Payments and the Concepts of Income
    1. R. F. Bennett, “Significance of International Transactions in National Income,” in Studies in Income and Wealth, Vol. VI. New York: The National Bureau of Economic Research, 1943.
    2. U. S. Department of Commerce, Income and Output, 1958 Supplement to the Survey of Current Business.
  7. The Theory of Income Transfers
    1. J. M. Keynes, “The Transfer Problem,” Economic Journal, Vol. XXXIX, No. 153, March 1929, pp. 1-7.
    2. B. Ohlin, “The Reparation Problem: A Discussion. I. Transfer Difficulties, Real and Imagined,” Economic Journal, Vol. XXXIX, No. 154, June 1929, pp. 172-78.
    3. J. M. Keynes. “The Reparation Problem: A Discussion. II. A Rejoinder,” Economic Journal, Vol. XXXIX, No. 154, June 1929, 179-82.
    4. J. Rueff, “Mr. Keynes’ Views on the Transfer Problem,” Economic Journal, Vol. XXXIX, No. 155, September 1929, pp. 388-99.
    5. B. Ohlin, “Rejoinder to J. Rueff,” Economic Journal, Vol. XXXIX, No. 155, September 1929, pp. 400-04.
    6. J. M. Keynes, “Reply to J. Rueff,” Economic Journal, Vol. XXXIX, No. 155, September 1929, pp. 404-08.
    7. L. A. Metzler, “The Transfer Problem Reconsidered,” Journal of Political Economy, Vol. L, No. 3, June 1942, pp. 397-414.
    8. H. G. Johnson, “The Transfer Problem and Exchange Stability,” Journal of Political Economy, LXIV, No. 3, June 1956, pp. 212-25, Republished in International Trade and Economic Growth. London: George Allen and Unwin, Ltd., 1958, Chapter VII.
    9. L. A. Metzler, “Flexible Exchange Rates, The Transfer Problem, and the Balanced-Budget Theorem,” Rivista Internazionale di Scienze Economiche e Commerciale, Anno XIII, No. 4, April 1966, pp. 301-18. Republished in Essays in Honor of Marco Fanno, Vol. II, Tullio Bagiotti, editor, edizioni cedam, Padova, 1966, pp. 449-76.
  8. Evolution of the International Monetary System
    1. Randall Hinshaw. Toward Currency Convertibility. Princeton University, Essays in International Finance, No. 31, 1958.
    2. Robert Triffin, Europe and the Money Muddle, New Haven: Yale University Press, 1957.
    3. Charles P. Kindleberger. The Dollar Shortage, Cambridge, Mass.: The Technology Press, New York: John Wiley and Sons, Inc., 1950.
    4. Robert Triffin, “The International Monetary Position of the United States,” in The Dollar in Crisis, Seymour E. Harris, editor, New York: Harcourt, Brace and World, Inc., 1961.
    5. Hal B. Lary, Problems of the United States as World Trader and Banker, Princeton: Princeton University Press for the National Bureau of Economic Research, 1963.
    6. Robert Triffin, The Evolution of the International Monetary System: Historical Reappraisal and Future Perspectives. Princeton Studies in International Finance Section. Princeton University, 1964.

Source: Duke University. David M. Rubenstein Rare Book and Manuscript Library. Economists’ Papers Archive. Lloyd Appleton Metzler Papers, Box 9, Folder “The Dust Proof File”.

__________________________

L. A. Metzler

ECONOMICS 370
FINAL EXAMINATION
WINTER, 1971

Answer all questions

  1. It is frequently said that forward purchases and sales have a two-fold effect upon the assets and liabilities of the bank making the forward transactions. When a bank makes a forward purchase, for example, it is entitled to receive a given amount of foreign currency some time in the future and this right to receive foreign currency constitutes an asset. On the other hand, when the bank receives the foreign currency, it is obligated to pay a given amount of its own currency, and exchange, and this obligation constitutes a liability. Conversely, when the bank makes a forward sale, it is obligated to deliver a given amount of foreign currency some time in the future and such an obligation constitutes a liability. But when the bank delivers the foreign currency, it is entitled to receive a given quantity of domestic currency to complete the transaction, and this receipt constitutes an asset.
    1. In view of the two-fold effect of both forward sales and purchases, how can you justify the inclusion of forward sales as liabilities and the inclusion of forward purchases as assets in an account of the bank’s foreign currency position?
    2. If it is actually true that both sales and purchases increase assets and liabilities by the same amount, what does this imply with respect to the bank’s ability to put itself in a closed position by means of operations on the forward exchange market?
  2. The table, below, presents the yield on 90-day Canadian treasury bills (rc), the yield on 90-day U.S. treasury bills (rus), the 90-day forward exchange rate (FR), and the spot exchange rate (SR) for Canadian currency. The yields for both Canadian and United States bills are stated on an annual basis and the exchange rates represent the price, in United States dollars, of one unit of Canadian dollars. The table covers various periods of time, from A to I.
Period of time Yield on
90-day Canadian treasury bills (rc)
Yield on
90-day
U.S.
treasury bills (rus)
90-day forward exchange rate (FR) Spot exchange rate (SR)
A .05 .06 1.0100 1.000
B .06 .05 1.0050 1.000
C .06 .04 1.0025 1.000
D .07 .03 0.9975 1.000
E .05 .04 0.9900 1.000
F .04 .05 0.9950 1.000
G .02 .04 0.9975 1.000
H .02 .06 1.0025 1.000
I .01 .05 2.0050 2.000

On the basis of the information given in the table above, you are asked to:

    1. Indicate which periods are periods of short-term potential capital outflow (O), and which periods are periods of short-term capital inflow (I) from the point of view of U.S. banks.
    2. Prepare a table showing the profits or losses on security transactions, the profits or losses on currency transactions, and the net outflow or inflow margin for all time intervals from A to I.
    3. Show what transactions a U.S. bank would make in carrying out a short term covered capital outflow and what transactions the bank would make in carrying out a short term capital inflow. From this information, show that arbitrage activities of the U.S. bank always influence rc, rus, FR, and SR in such a way as to put the market values back on the interest parity line.
    4. Comment on the stabilizing effects of interest arbitrage.
  1. Professor Alexander maintains that devaluation will be ineffective unless fiscal measures are taken to control spending.
    1. Discuss Alexander’s argument.
    2. Is it also applicable against a system of flexible exchange rates? Explain.
  2. Country A and Country B are trading with each other under a system of flexible exchange rates. Country A makes a unilateral transfer of t currency units, payable in the currency of B. On the assumption that both countries balance their budgets, prove that factor income remains unchanged in both countries while net output rises in A and falls in B. Give both an algebraic and a commonsense explanation of these results. What bearing do these results have on the controversy between Keynes and Ohlin concerning German reparations?
  3. The success of the Canadian experiment with flexible exchange rates is frequently given as an argument for the introduction of flexible exchange rates as a means of eliminating the deficit in the U.S. Balance of Payments.
    1. Would Alexander’s absorption principle apply to this situation?
    2. Can you see any reason why the comparison between Canada and the United States might be inappropriate?

Source: Duke University. David M. Rubenstein Rare Book and Manuscript Library. Economists’ Papers Archive. Lloyd Appleton Metzler Papers, Box 9, Folder “Econ 371 [sic] Reading List.”

Source Image: Posting by Margie Metzler on the Metzler Family Tree at the genealogical website, ancestry.com.

Categories
Harvard Statistics Suggested Reading

Harvard. Short Bibliography on Social Statistics for “Serious-minded Students”, Ripley, 1910

 

In 1910 Harvard published a total of 43 of short bibliographies in the collection “Social Ethics and Allied Subjects”, about half of which were dedicated to particular topics in economics and economic sociology. The project was coordinated by Plummer Professor of Christian Morals, Francis G. Peabody.

Part I, Chapter 7 “Social Statistics” was the first of economics professor William Z. Ripley’s three contributions to the collection. Most items have been conveniently linked to internet archival copies for this post.

Previously posted bibliographies from Peabody’s “Social Ethics and Allied Subjects”:

Economic Theory by Professor Frank Taussig

Taxation by Professor Charles J. Bullock

Trade Unionism by Professor William Z. Ripley

Social Insurance by Dr. Robert Franz Foerster

Economics of Socialism by Professor Thomas Nixon Carver

Strikes and Boycotts by Professor William Z. Ripley

Socialism, Family and Religion by Dr. Ray Madding McConnell

______________________

7. SOCIAL STATISTICS

William Z. Ripley

The principal source for the United States is the Federal Census,” taken decennially since 1790. The later ones are summarized in special volumes, that for 1900 being known as the Supplementary Analysis.” A still briefer Abstract” is also published. The Supplementary Analysis” is, however, more serviceable, inasmuch as it offers in the text a critical examination of the figures, setting forth the limitations upon their use. A Statistical Atlas” also accompanies each census since 1870, offering the results in graphic form. Special reports” on a variety of topics are also issued. Among these since 1900 may be mentioned, A Century of Population Growth,” Washington, 1909; Marriage and Divorce,” 1867–1906, two parts [Part One Summary, Laws, Foreign Statistics; Part Two, General Tables], Washington,1909, and Statistics of Cities.” “Mortality Statistics” for all states having registration laws are also compiled and published annually. This is the standard reference for all states and minor subdivisions.

Several of the Commonwealths, notably Massachusetts, take censuses on the intervening quinquennial years, some of them containing more detailed information than is afforded by the federal census. The United States Statistical Abstract,” published annually [1878-2018], is often serviceable.

For foreign countries, the Statistical Abstract for Great Britain” and also “The Statistical Abstract for the Principal and other Foreign Countries,” will serve as convenient and accurate handbooks. Some of the leading newspapers, notably the New York Tribune [1856-1914] and the World, [1868-1925], publish Almanacs,” containing a wide range of material gathered from official publications. The Statesman’s Year Book” [1876-1923] is a valuable source. The most elaborate scientific collection, issued annually and covering a very wide range of social data, is, G. Sundbärg: Aperçus statistiques internationaux.” Stockholm: Imprimerie Royale. Since 1906 this has been published in French.

Among scientific serials which are invaluable to students of social statistics, not only for their separate articles, but for their reviews of current literature, should be included: Quarterly Publications of the American Statistical Association [1888-1921] and Journal of the Royal Statistical Society.

The following standard treatises are of fundamental importance, containing not only positive statistical data, but critical examination as to their value:

Mayo-Smith, Richmond. Statistics and sociology. New York: The Macmillan Company, 1895, pp. xvi, 399.

Mayo-Smith, Richmond. Statistics and economics. New York: The Macmillan Company, 1899, pp. 467.

These volumes of material up to the date of their publication are most valuable. They are, however, less ample in American material than for foreign countries.

Bailey, William B. Modern social conditions. New York: The Century Company, 1906, pp. 377.

This treatise is confined principally to the field of vital statistics and problems of population.

Bowley, Arthur Lyon. Elements of statistics. London: P. S. King & Son, 1901, pp. 328.

The most scientific treatise extant as to statistical method, but less serviceable as a collection of positive data. [First edition (1901); Second edition (1902); Third edition (1907); Fourth edition (1920); Fifth edition (1926); Sixth edition (1937)]

Wright, Carroll D. Outline of practical sociology. New York: Longmans, Green & Co., 1899, pp. XXV, 431.

This obviously does not include data from the census of 1900.

Newsholme, A. Elements of vital statistics. Third edition. London: Swan, Sonnenschein & Co., 1899, pp. xxiv, 326.

The standard authority for this special field.

Walker, Francis A. Discussions in economics and statistics. 2 vols. New York: Henry Holt & Co., 1899, pp. iv, 454, 481. [Volume 1, Finance and Taxation, Money and Bimetallism, Economic Theory; Volume 2, Statistics, National Growth, Social Economics]

Weber, Adna F. The growth of cities in the nineteenth century. New York: Columbia University Studies, 1899, pp. xiv, 497.

This work is a convenient source book for all statistics of cities.

Schnapper-Arndt, Gottlieb. Sozial-Statistik. Leipzig: W. Klinkhardt, 1908, xxii, 642 S.

This book offers a readable account of statistical data, mainly for European countries.

Mayr, Georg von. Statistik und Gesellschaftslehre. 3 Bde. Freiburg i. B.: J. C. B. Mohr, 1895-1909, 202, 486, 260 S.  [Volume 1, Theoretische Statistik; Volume 2, Bevölkerungsstatistik; Volume 3, Sozialstatistik]

The most elaborate treatise, serviceable mainly to specialists. It contains a wonderfully complete bibliography in all languages, arranged under separate headings, which may be of use in looking up special topics.

Source: A Guide to Reading in Social Ethics and Allied Subjects. Lists of books and articles selected and described for the use of general readers by teachers in Harvard University. Cambridge, Massachusetts (1910), pp. 32-34.

Image Source: Harvard University Archives. William Zebina Ripley [photographic portrait, ca. 1910], J. E. Purdy & Co., J. E. P. & C. (1910).

Categories
Economists Gender Harvard Smith

Radcliffe/Harvard. Economics Ph.D. alumna Eleanor Martha Hadley, 1949

 

This addition to our intermittent series “Get to know an economics Ph.D. alumna” is dedicated to the Radcliffe expert on Japanese industrial organization whose government career prospects were blocked for some seventeen years after she had been denied a security clearance. This was the work of General MacArthur’s “lovable fascist”, Maj. Gen. Charles Willoughby [a bit of backstory to Willoughby’s purge of Hadley is provided below]. 

Incidentally, The Diplomat (January 27, 2019) ran a story about Charles Willoughby with the title “Is This the Worst Intelligence Chief in the US Army’s History?” Plot spoiler: He and his boss MacArthur share the blame for the Yalu River disaster for the United Nations forces.

______________________________

Eleanor Hadley’s Memories of Radcliffe and Harvard

…Being somewhat at loose ends upon my return from Japan in the spring of 1940, I ended up attending the University of Washington in Seattle for the academic year 1940-1941. There I took courses in economics and in the Far East Institute, and found the Japanese-language instruction a great improvement over that I had known in Tokyo.

Finally pulling myself together, I decided to embark on a Ph.D. program in economics. It was not that economics was my favorite subject; but I assumed that I needed to build on my undergraduate work, which had been a degree in politics, economics, and philosophy. It had not occurred to me that I could choose any subject I wanted. Much as I loved philosophy, I did not see taking a graduate degree in it. Between politics and economics, I believed that the latter favored classroom discussion; and I thought that I could do reading about politics on my own.

I wanted to attend Radcliffe College; but the problem was how to finance it. Then, out of the clear blue sky, my great-aunt in Honolulu, who had lost her sister earlier that year, said that if I would spend the summer with her there, she would make it financially possible for me to enter Radcliffe that fall. I could scarcely believe my good fortune.

In Honolulu that summer, one of my America-Japan Student Conference friends from the University of Washington was enrolled in the U.S. Marines’ Japanese-language course. The Marines had decided that their service required some competency in the Japanese language. One thinks of Marines as ramrod straight. In fact, the course was so strenuous that every week my friend’s shoulders were slightly more rounded.

Cambridge, Massachusetts

Thanks to my great-aunt, I entered Radcliffe in the fall of 1941 to begin a Ph.D program in economics. I remember that, so splendidly ignorant as I was of the college’s setup, I said to the dean of graduate students with a catalogue in my hand, “I see Harvard faculty listed here, but where is the Radcliffe faculty?” She replied: “Don’t you know that we are medieval? There is no Radcliffe faculty.”

Radcliffe College, both graduate and undergraduate, consisted of students and administrators but no faculty. Harvard did not admit women; Radcliffe existed to provide a Harvard education to women. For undergraduate students, Harvard faculty crossed the Cambridge commons and delivered an identical lecture to the women. For graduate courses, women crossed the commons to Harvard Yard and attended classes with the men. And though Radcliffe’s graduate final exams were identical to what the men took, they were administered in Radcliffe buildings. It was the advent of World War II, and the consequent scarcity of both faculty and students, that disrupted the pattern of duplicative lectures at the undergraduate level. Graduate women were first admitted to Harvard classes in September 1941, just three months before Pearl Harbor.

The fall of 1941 was also the first time Radcliffe graduate women were permitted to sit in the reading room of Widener Library, then the main library for Harvard students. We sat at one designated table, and this table bore signs that could be seen from whatever angle one approached it, announcing, “This table reserved for Radcliffe students taking graduate courses.” Previously, female graduate students had been permitted to sit only in a room separate from the reading room about twenty by twenty feet in size.

In all, the college informs me, there were eighteen students in economics in 1941-1942, and fourteen in 1942-1943. I believe that most of these must have been in the Ph.D. dissertation stage, because when it came to graduate students that one actually saw in classes or in the dormitory, there were only three or four of us. The college speaks of total enrollment in the graduate school of the year 1941-1942 as having been 241; it was 253 in 1942-1943.

Much of the time in classes with the Harvard men I sat petrified with fear. The men were so knowledgeable-that is, most of the time. A number of them had previously held positions bearing on the topics under discussion. Economics was not a Mills College point of strength. If I had entered graduate school in philosophy I would have felt comfortable, but not in economics. Although Keynes’ General Theory of Employment, Interest and Money was published in 1936, it had not made the eco- nomics department of Mills College by the time I graduated in 1938. And in Japan, of course, I had had no exposure to the latest work of Western economists. Thus the General Theory was brand-new to me while familiar to most of my colleagues.

Graduate study in economics in the early 1940s was far from being a purely academic exercise. Students and faculty alike were in constant debate about how to apply what they knew to the urgent issues of the day. In the face of a catastrophic depression in the United States, where in 1933 one-fourth of the labor force was unemployed, Herbert Hoover had seen solutions in smaller government expenditures and balanced budgets. Franklin Delano Roosevelt, although campaigning in 1932 for balanced budgets, rapidly changed his mind once he was in office and saw solutions in terms of government expenditure in excess of tax income. The role of government in the economy was the defining point of a New Deal Liberal. Republicans were afraid of a large government role; Democrats were not. The difference was accounted for by differing views of market forces: would they always equilibrate demand and supply or were there times when they would be incapable of doing so?

The economics department of Harvard University was superb in the time period I was there. Among the faculty were Joseph Schumpeter (economic, thought, capitalism and socialism), Alvin Hansen (business cycles), Gottfried Haberler (international trade), Sumner Schlichter (labor), Wassily Leontief (input-output), Alexander Gerschenkron (economic development), John Williams (money and banking). The problem with a small institution such as Mills was that the department tended to depend on one individual. As one example, Harvard’s economics department was divided on the subject of Keynes, which made for great liveliness.

While the department had its share of outstanding men, it also had its share of prejudices. The faculty had only one Jew, Seymour Harris, and one was “enough.” Paul Samuelson, a few years ahead of me, would find no teaching offer from Harvard. Accordingly, he went slightly downriver and accepted MIT’s offer. In retrospect, how the department must have rued this decision.

The department, at this time, did not like the master’s degree, so in consequence the difference between the master’s and doctoral degree was the dissertation. As explained by the college’s official register: “The general examination for the Ph.D. is the same examination as for the Master’s degree.”

At a party I was introduced to Mrs. Chamberlin, a Frenchwoman who was the wife of Edward Chamberlin, the well-known Harvard economist who had already published his influential Monopolistic Competition. She asked me what field I was in and I said economics. Her wonderful reply was “Well, you don’t look like one,” which I regarded as a compliment.

I continued to study the Japanese language, this time using a text that included grammar. It was prepared by Sergei Elisseeff and Edwin Reischauer. Elisseeff had gone to Japan from St. Petersburg after the Russo-Japanese War, becoming the first Westerner to graduate from Tokyo Imperial University. After the Communist takeover of Russia in 1917, he emigrated to France where he taught Japanese and Chinese at the Sorbonne, and then from 1934 to 1960, at Harvard. Reischauer had grown up in Japan, where his parents were missionaries. After his graduation from Oberlin College, he entered Harvard as a graduate student in the fall of 1935.

It was almost impossible to study Japanese in Widener Library during that fall of 1941 without interruption. Anyone passing the table reserved for Radcliffe graduate students in the reading room and seeing the unusual script had to stop and inquire what it was.

That fall I attended my first “House” dance at Eliot House on the river as the guest of John Lintner, an economist who was to become a junior fellow (a much sought-after distinction) at Harvard with a specialization in public finance. I had two astonishing experiences. One was learning that one had to think of the outside temperature before adding a corsage to one’s outfit; if one were nonchalant in the late fall, winter, or early spring, the cold might do it in. Second, coming from the West Coast, I was flabbergasted to see the whole inner wall of the dining room (converted to a ballroom) covered to a height of six to eight feet with cases of sherry, bourbon, scotch, and gin. On the West Coast at that time, one could not even sell liquor within a mile of a public educational institution. At Mills College in the 1930s it was a “sin”to have even beer on the campus. Imagine that many cases of liquor on campus! Unbelievable.

Inasmuch as so many Radcliffe graduate students were from other parts of the country as well as from abroad, the dean of the graduate school, Mrs. Cronkite (it was a Harvard affectation to drop the “Dr.”) arranged sight-seeing tours of nearby New England towns for us on Saturday afternoons. These had come to a halt soon after December 7, when gasoline conservation became necessary.

Everyone who was beyond infancy in 1941 remembers where he or she was on December 7. I was starting Sunday dinner (at that time served by maids) in the Radcliffe graduate dormitory at one o’clock. We had just begun to eat that Sunday when someone reported hearing a radio report of a Japanese attack on Pearl Harbor. Perhaps more than others, I was incredulous that Japan would attack the United States. Japan, of course, had all sorts of differences with the United States, but for it to take us on in armed conflict seemed unbelievable. It was clearly foolhardy, but Japan’s military apparently reasoned that, owing to Japan’s alliance with Germany, their attack would draw the United States into the European war. And a two-front war they believed Japan could win.

My father used to say that one positive feature of war is that it brings persons and nations in touch with reality. Japan’s military discovered that reality was different from what they had imagined, and so did the United States. In those months following Pearl Harbor, when every news report brought word of Japanese victories and our defeats, respect for Japan’s military prowess increased a great deal.

Some of my Radcliffe friends went down to Washington, D.C., during the summer of 1942, on completing the 1941-1942 academic year. With the United States at war, everyone was anxious to contribute to the mammoth effort our country was facing, and to become involved without delay. I, however, held off, because I wanted to get through my Ph.D. “general” examinations first. The thought of taking exams on course work done years earlier was daunting to me.

Under the system then in effect in the Economics Department, one presented oneself in six fields, four of which were examined in the “general” examination. One of the six one was allowed to “write off” — that is, fulfill the requirement with course work. I did that in statistics. My four fields for the general exams were theory, money and banking, international trade, and economic history. The Ph.D. requirements also included qualifying in two languages — normally, French and German. Harvard granted my petition to use Japanese as my second language, my first being French. I don’t believe the Economics Department had ever had to consider such a request before. Having passed all of these exams in the summer of 1943, I then went down to Washington that fall.

The sixth field was the dissertation field, in which one took the separate, “special” exam. At this time, when I took the other general exams, I had in mind to make public finance my “special” dissertation field. But I later changed to industrial organization as a result of my State Department and MacArthur staff positions. It would be in industrial organization, therefore, that I eventually took the final exam after completing my dissertation, entitled “Concentrated Business in Japan,” in 1949.

One major legacy of my Radcliffe years is a lifelong friendship with a fellow economist from Peking, Shu-chuang Kuan. It was in Cambridge in that first fall of 1941 that I met Shu-chuang. Like me, she was beginning the program in economics, and we became close friends. Our friendship has lasted to the present day, although it was interrupted for a long time by events beyond the control of either of us; nowadays, we speak regularly by telephone although we live on opposite coasts of the United States.[1]

My second year at Radcliffe, I became a “head of house” of one of the smaller dormitories. Radcliffe used graduate students for that role rather than having older women as “house mothers,” as was done in a great many colleges. In 1942-1943 we were all graduate students with one exception, an older woman from Concord, Massachusetts. It happened that she invited me to join her on a particular Friday evening. Instead of simply saying that I had a previous engagement, I said I had an invitation to the waltzing party — an event considered of great significance among the “socially acceptable” persons of Boston. Her memorable reply was, “My dear, and only your second season!”

It was customary in that period for female students to wear skirts. That was the only attire considered appropriate for attending class. The Radcliffe dorms where we lived (there was no mixing at that time) were roughly a mile from the Harvard Yard. To walk that mile with legs clad only in stockings when the weather was well below freezing was so painful that I occasionally had to stop at the Commodore Hotel on the way to thaw out.

To me, a New England spring was an astonishing experience. February came and February went. March came and March went. It was not until April that the grass began to turn green and there were crocuses. In Seattle, as in Tokyo, spring begins early. In Seattle one can have pussywillows and crocuses in February, as well as the first blooms of the camellia. In New England, May is one grand riot as the season makes up for its slow start. Everything bursts into leaf and bloom at the same time.

A Year at the OSS

While still in Cambridge I had been recruited by Charles B. (Burton) Fahs [2] for a position in the Research and Analysis Branch, Far East, of the Office of Strategic Services (OSS), which was one of what came to be five competing intelligence groups in Washington, D.C. (The other four were the Army, Navy, and Foreign Economic Administration [FEA] organizations, and subsequently the Air Force Intelligence group.) Roosevelt favored competition in his government.

I entered the OSS as a P-3, the equivalent of today’s government service classification of GS-9. I was put to work assessing the significance of Japan’s wooden-shipbuilding program, which Japan had begun in response to the shortage of steel.

Even though it was conventional in that period to dislike Washington, I loved the city from the moment I arrived. But where would I live? Washington was still suffering an acute shortage of housing. I located an apartment, but it was still under construction. A Radcliffe friend, Ruth Amande (Roosa), said that I might join friends with whom she was sharing a house, and that is what I did for six weeks or so.

That is also how I came to know Ralph Bunche, for his secretary was part of the same household. Bunche was at the OSS too, but in another part of the organization. In 1944 he was invited to become an assistant secretary of state, the first African American to be so invited, and transferred over from the OSS to the State Department. Subsequently, Bunche won the Nobel Peace Prize in 1950 and was undersecretary of the United Nations from 1955 to 1971. His B.A. was from UCLA in 1927; his Ph.D., from Harvard in 1934. We had a friendly relationship, although not a close one, during the period when we were both at the State Department.

Source: Eleanor M. Hadley. Chapter 2 “Radcliffe College and Washington, D.C.” from Memoir of a Trustbuster: A Lifelong Adventure with Japan. Honolulu: University of Hawaii Press, 2003. Pages 42-48.

______________________________

Radcliffe Ph.D. Awarded June 1949

Eleanor Martha Hadley, A.M. [Radcliffe College, 1943]

Subject, Economics. Special Field, Business Organization and Control.
Dissertation, “Concentrated Business Power in Japan”.

Source: Radcliffe College. Reports of Officers Issue 1948-49 Session. Official Register of Radcliffe College, Vo. XV, No. 6 (December, 1949, p. 21.

Note: B.A. Mills College (Oakland, CA), 1938

______________________________

Career

1943 – 1944. Research analyst , Office Strategic Services. Washington, D.C.

1944 – 1946. economist, Department State. Washington, D.C.

1946 – 1947. economist, GHQ-Supreme Command Allies Pacific. Tokyo, Japan.

1950 – 1951. staff member, President Truman’s Commission Migratory Labor. Washington, D.C.

1956 – 1965. associate professor, Smith College. Northampton, Massachusetts.

(ca 1963-64 Fulbright Fellowship to Japan)

1967 – 1974. economist , United States Tariff Commission. Washington, D.C.

1972 – 1984. professorial lecturer, George Washington University. Washington, D.C.

1974 – 1981. group director international division, General Accounting Office. Washington, D.C.

1986 – 1994. visiting scholar, University Washington, Seattle, Washington.

Source:

https://web.archive.org/web/20211124081228/https://prabook.com/web/eleanor_martha.hadley/695543

__________________________________

Books

Hadley, Eleanor. Antitrust in Japan. Princeton: Princeton University Press, 1970.

_____________. Japan’s Export Competitiveness in Third World Markets. Georgetown: The Center for Strategic and international Studies, 1981.

_____________. Memoir of a Trustbuster: A Lifelong Adventure with Japan. Honolulu: University of Hawaii Press, 2003.

______________________________

Testimony before the Subcommittee on Antitrust and Monopoly, Committee on the Judiciary, U.S. Senate. July 29, 1964 pp. 147-161.

__________________________________

Obituary

Eleanor Hadley spent her life standing up to oppression, dies at 90

By Sara Jean Green The Seattle Times, June 6, 2007

Eleanor Hadley rarely talked of her experiences as a young American woman tasked with democratizing the economy of post-World War II Japan, preferring instead to discuss politics and policy with guests who would stop by her Normandy Park home for an intellectual chat and a cup of tea.

She’d fought a 16-year battle to clear her name after she was secretly added to a McCarthy-era blacklist, but Ms. Hadley was never bitter — though she was plenty indignant in the grand, gutsy way that family and friends say she reacted to any injustice or abuse of power.

Ms. Hadley, who dedicated her life to academia and government service, died from natural causes at Seattle’s Swedish Medical Center on Friday (June 1). She was 90.

A 1986 recipient of Japan’s Order of the Sacred Treasure for meritorious service, Ms. Hadley was finally persuaded by a group of admirers to pen her autobiography, co-authoring “Memoirs of a Trust Buster: A Lifelong Adventure with Japan” in 2003.

“She was one of the very few women in a leadership position during the occupation” of Japan, said professor Kenneth Pyle, of the University of Washington’s Henry M. Jackson School of International Studies. “It was rare because it was a man’s world. … She was a very independent and assertive woman in an environment that did not encourage that.”

Eleanor Martha Hadley was born July 17, 1916, in Seattle, graduating from Franklin High School in 1934. Her father, Homer Hadley, an engineer, first conceived the idea of a concrete floating bridge across Lake Washington; and her mother, Margaret Hadley, was a pioneer in preschool education and the education of children with disabilities. Her brother Richard Hadley, who died in 2002, was a prominent Northwest land developer.

Ms. Hadley attended Mills College in Oakland, Calif., and was selected for a student fellowship at Tokyo Imperial University, said her nephew, Robert Hadley, of Normandy Park. From 1938 to 1940, she traveled extensively in Japan and China, becoming one of the first Westerners to visit Nanjing after the Japanese military massacred 150,000 to 300,000 Chinese in that city.

“She went to Japan a pacifist but came back from the whole experience with an understanding that there are times you have to stand up to horrible regimes,” her nephew said.

She returned to the U.S. to pursue her doctorate in economics at Harvard-Radcliffe University but was recruited in 1943 by the U.S. State Department to work as a research economist focusing on Japan.

At the end of the war, Ms. Hadley — then 31 — was asked to join Gen. Douglas MacArthur’s staff in Tokyo, where she worked to help break the zaibatsu, the powerful industrial and financial combines that dominated Japan’s economy.

Ms. Hadley returned to Harvard in 1947 to complete her doctorate and planned to join the newly created Central Intelligence Agency, Robert Hadley said. But the CIA job offer — and her security clearance — were mysteriously withdrawn. She didn’t learn until years later that she’d been labeled a Communist and was blacklisted by Maj. Gen. Charles Willoughby, MacArthur’s conservative chief of intelligence.

She later became a professor at Smith College in Massachusetts and George Washington University in Washington, D.C. For 16 years, she worked to clear her name and finally prevailed after Sen. Henry “Scoop” Jackson from Washington intervened on her behalf. She went to work for the U.S. Tariff Commission (now the International Trade Commission) and General Accounting Office, returning to the Seattle area after her retirement in 1984.

In addition to her nephew Robert Hadley, Ms. Hadley is survived by her nephew Scott Talley of Colorado Springs, Colo.; and nieces Alisa Scharnickel, of Arlington, and Lisa Hadley, of Honolulu.

Source: Web-archive copy of the Hadley obituary.

__________________________________

Backstory: “The Purge of Hadley”

Source: Thiry, Martin. Chapter 2 “Eleanor Hadley: Anti-Trust in Occupation Japan”, Domestic Politics and Foreign Policy, US. 1946-1954: Three at the Intersection and What it Wrought. M.A. (History) Thesis, University of Hawai’i (August, 2007),  pp. 56-58.

…Hadley left Japan in September of 1947 to finish her doctorate at Harvard. The years ahead were black ones. She was recruited by the nascent CIA for analyst work, but she was unable to get a security clearance. She was turned down by several government agencies. She worked on the fringes of the Washington bureaucracy until 1956 when she got a job teaching at Smith College. In 1965 Henry “Scoop” Jackson took up her case. He was able to track down the retired Whitney who joined in the effort to clear her name. There was nothing in the GHQ-SCAP files to suggest disloyalty. At the end of 1966, through some machinations on the part of Jackson, Hadley was finally given her clearance. The years of banishment had been long. The climate in Washington had been harsh to one “under a cloud.” She had been blacklisted from before the coming of McCarthy and remained so long past his demise. Dean Rusk, an old college teacher, even refused his help.[214] “I was afraid to get a book out of the library (in those days)… [I] was miserable going through it,” Hadley remembers.

The mystery of why she had been blacklisted was eventually resolved. Major-General Charles Willoughby had been the head of SCAP’s Military Intelligence Section. Later an advisor to General Franco, he maintained extensive surveillance on Japanese radicals as well as reporting critically on American reformers within SCAP itself.[215] Willoughby was an ultraconservative and controlled censorship for SCAP[216] He had a personal rivalry with Whitney, which may have accounted for some of Government Section’s pursuit of reform: Whitney knew Willoughby would hate it. Willoughby brought extreme right-wing views and a Prussian bearing to his job. MacArthur called him “my lovable fascist.”[217]

Willoughby’s papers were declassified in 1975,[218] including a report on “Leftist infiltration into SCAP.” Hadley was mentioned. The concern was that she was dating a journalist. “Her relative immaturity… suggest the possibility.… [of] being exploited by leftists.”[219] Hadley shared her thoughts on Willoughby:

Hadley: Politics shaped his job. He was security. After five years of Mr. Bush we know how far security can be pushed. I enjoyed having dinner with foreign correspondents, US and European. It’s possible I said something one night. I was never informed about that.

Author: What were the specific accusations against you?

Hadley: No specific accusations whatsoever. It was all done very quietly. The black ball consisted of telling people in D.C. that I was doubtful. I “might” have spilled the beans, I “might” have been indiscreet, I “might” have indicated SCAP direction to foreign correspondents. All “might have’s”- Willoughby’s wonderment.

Author: Would you have led a different life if you had not been black balled?

Hadley: 17 years out of [one’s] most productive makes a dent.[220]

Without access to Willoughby’s files there is no way to confirm Hadley’s story. Still, I find it plausible. The forces around her, the clash of competing ideologies within SCAP and the US political scene, worked greater effects. I do not render judgment on the issue of zaibatsu dissolution. I simply do not know enough about it. What I do feel comfortable arguing is that the clash of ideologies within SCAP and beyond about zaibatsu dissolution became increasingly Orientalized the more it entered into the discourse of US domestic politics. And the more entrenched it became in US domestic politics the more Orientalized it became. The power of this convergence was more than enough to wreck Hadley’s career in government…

[214] Hadley, Memoir of a Trustbuster, 121-145.

[215] Bailey, Paul. Postwar Japan: 1945 to the Present. Cambridge: Blackwell Publishers (1995), 31.

[216] Dower, John. Embracing Defeat: Japan in the Wake of World War II. New York: W.W. Norton and Compnay (1999), 406.

[217] Schaller, Michael. Douglas MacArthur: The Far Eastern General. Oxford: Oxford University Press (1989), 121.

[218] His papers are on file at the MacArthur Memorial in Norfolk, VA, but not on-line. http://www.macarthurmemorial.org/archives_record.asp (accessed 1/30/07).

[219] Hadley, Memoir of a Trustbuster, 146.

[220] Eleanor Hadley, “Telephone Interview With Eleanor Hadley 1/25/07” (Honolulu-Seattle).

__________________________________

Image Source: From the website World War II Database. Archived copy at web.archive.org.

Categories
Economics Programs Graduate Student Support Harvard Undergraduate

Harvard. Economics Chairman’s Report to the Dean. Harris, 1956

 

The previous post provided transcriptions of the annual reports to the Dean by the chairman of the economics department from 1932 through 1941. This post skips ahead to the middle of the 1950s to give us a glimpse of the post-war Harvard economics department. Seymour Harris’ big take-aways from his 45 year survey of undergraduate and graduate economics courses taught by Harvard economics faculty: (i) “the proportion of undergraduate courses given by full professors has fallen from 75 to 35 percent” and (ii) “graduate courses are relatively 5 times as numerous as they were in 1909-10.” (from July 3, 1956 cover letter to Dean McGeorge Bundy that accompanied the report transcribed below).

It is also interesting to note that the economics department’s continues to plead for more funds to compensate it for “…about one half the teaching burden of the G.S.P.A. and students in the G.S.P.A. account[ing] for about one third of all the graduate students in economics (on a full-time basis)…”. Harris wrote this report two decades after the Graduate School of Public Administration had opened for business.

____________________________

CONFIDENTIAL

June 30, 1956

Report to the Dean of the Faculty for the Academic Year 1955-56
by Seymour E. Harris, Chairman of the Department of Economics

Contents

Undergraduate Instruction

  1. More Mature Staff for Economics 1.
  2. Contents of Economics 1.
  3. Staff Meetings of Economics 1.
  4. Lectures in Economics 1.
  5. Economics Tutorial.
  6. High Honors Concentrators.
  7. Seminars for Honors Graduates.

Allocation of Resources

  1. Enrollment of Undergraduates in Graduate Courses and Vice Versa.
  2. Increase in the Number of Undergraduate Courses, 1909-10 to 1955-56.
  3. Increase in the Number of Graduate Courses, 1909-10 to 1955-56.
  4. Table 1 – Distribution of Courses by Academic Rank, 1909-10 to 1955-56.
  5. Table 2 – Courses Given by Faculty, 1909-10 to 1955-56, by Rank.
  6. Table 3 – Percentage of Courses, Undergraduate and Graduate.
  7. The Increased Importance of Graduate Instruction.
  8. Reduced Undergraduate Instruction by Higher Ranking Members of Faculty.
  9. Ibid., Statistical Summary.
  10. Number of Faculty by Rank.

Relations with G.S.P.A.

  1. Teaching Responsibilities of Economics Department in G.S.P.A.
  2. Contributions of G.S.P.A. to Economics Department.
  3. Overall Consideration of Number of G.S.P.A. Seminars.

Library Problems

  1. Library Problems.

Fellowships

  1. Inadequate Fellowships.
  2. Campaign for Additional Money.
  3. Outside Fellowships.

Research and Personnel Problems

  1. Competition of Research Fellowships for Potential Teachers.
  2. Research Projects.
  3. Financing of Pay of Director of Research Projects.
  4. Small Research Grants.
  5. Secretarial Help.
  6. Personnel Changes.
  7. Honors, etc.

 *  *  *  *  *  *  *  *  *  *  *  *  *

Undergraduate Instruction

The Department is especially concerned with the problem of undergraduate instruction. Confronted with a trend away from economics the country over (see my Memo to the Alumni of the Harvard Graduate School in Economics, May, 1956, p. 4) and the competition of an unusually able corps of undergraduate teachers in competing fields at Harvard and notably in history and government we are paying increased attention to our undergraduate instruction. In the last year we have taken the following steps:

  1. More Mature Staff for Economics 1. We are using a larger proportion of instructors and assistant professors in Economics 1. We expect that half the Economics 1 staff will consist of instructors and assistant professors in 1956-57 as compared with 20 per cent in 1955-56.
  2. Contents of Economics 1. We are revising Economics 1 for 1956-57. Economics 1 has become too technical. One advantage of increasing the average age of the staff is that the older men are less inclined to teach the highly technical economics they get in graduate courses. Probably less than 20 per cent of those enrolled in Economics 1 are, or are likely to become, concentrators in economics; and no more than 1-2 per cent will become economists. Our major responsibility is to give the student in Economics 1 relatively simple economic theory and relate it to the major issues of public policy. We intend to devote more time to integrating our economics with history and political science. Macroeconomics will continue to receive a major part of our attention, but less time will be given to the economics of the firm.
  3. Staff Meetings of Economics 1. The Chairman now meets with the Economics 1 staff for 1½ hours every 2 weeks and in every possible way is trying to make the teaching fellow and other junior members, who contribute so much time and enthusiasm to our teaching program, feel as though they are an important part of our department staff.
  4. Lectures in Economics 1. This year we doubled our lectures in Economics 1 — a lecture every other week. In these lectures we try to go over ground not covered in the readings and also incidentally to give the undergraduate an opportunity to listen to some of the top economists in the country. We are now not disposed to increase the number of lectures further but we shall continue the experiment. Of this I am convinced — lectures are not likely to be as important in Economics 1 as in the elementary course in government and history (Social Science). The undergraduate probably gets much more from discussions of economics in small sections than from lectures.
  1. Economics Tutorial. Tutorial in economics is not as good as it ought to be. We are wrestling with this problem. We intend to have more meetings of tutors and to impress upon them the importance of tutorial. At one of our Executive Committee meetings, we had a frank discussion with the seven masters and several senior tutors concerning our tutorial work. Our Junior tests, tied to house tutorial, seem to be working well. This year we prepared an extensive reading list for Sophomore tutorial; and next year we intend to integrate tutorial and Economics 1 more than in the past. We hope that tutorial in the second half of the Sophomore year will deal with some of the theoretical problems that will be excluded from Economics 1.
  1. High Honors Concentrators. This year we had periodic meetings with all first and second group men in economics. At these meetings (one evening every two weeks) we try to encourage discussions of important problems in the seminar manner.
  1. Seminars for Honor Graduates. Economics 100 and 102 are two new courses (to be introduced in 1956-57 and 1957-58) to be open to Junior and Senior honors students. They will be run on a seminar basis, limited in enrollment, and will be integrated with tutorial. The student will get an opportunity to deal with theoretical problems and their empirical counterpart.

Allocation of Resources

  1. Enrollment of Undergraduates in Graduate Courses and Vice Versa. Here are some tables which throw some light on the allocation of resources between undergraduate and graduate courses. Generally courses for undergraduates and graduates are taken primarily by undergraduates, and courses for graduates primarily by graduates. Hence, we assume that the courses for undergraduates and graduates are in fact courses for undergraduates and courses for graduates are in fact courses for graduates. (In the spring term 1956 the percentage of Arts and Science graduate enrollment in courses for undergraduates and graduates was 14 or 1 per cent of the 1181 enrolled in these courses; the enrollment of undergraduates in courses primarily for graduates was 10 of 482, or 2 per cent).
  2. Increase in the Number of Undergraduate Courses, 1909-10 to 1955-56. Table 1 reveals relatively unimportant changes in the number of courses for undergraduates; and the net change in the number of courses for undergraduates and graduates (in fact undergraduate courses) in the last 40-50 years has not been large. In 1909-10, there were 10½ undergraduate courses (inclusive of half courses for undergraduates and graduates and exclusive of bracketed courses); in 1955-56, there were 14½ of such courses.
  3. Increase in the Number of Graduate Courses, 1909-10 to 1955-56. It is especially in graduate courses that the rise has been spectacular. In 1909-10 there were 1½ graduate courses in Economics (exclusive of bracketed ones); by 1929-30, there were 11; by 1939-40, there were 12½ courses; by 1949-50, there were 21½ courses; and by 1955-56, there were 24. All these totals include half courses.
  1. Table 1 — Distribution of Courses by Academic Rank, 1909-10 to 1955-56*
    (Refers to Units of Full Courses)
  1909-10 1919-20 1929-30 1939-40 1949-50 1955-56
Rank U G U G U G U G U G U G
Full Prof. 8 1 3 7 4 ½ 7 7 ¼ 16 ¾ 8 15 ¼ 5 18
Assoc. Prof. 3 3 3 ¼ 1 ¾ 1 3 ¼ 3 2 ½
Asst. Prof. 1 ½ ½ 3 ½ 2 ½ 1 ½ 2 ½ 4 2
Instructor & Lecturer 1 3 1 1 ½ 1 1 ½ 1 3 3 2 ½ 1 ½
Total 10 ½ 1 ½ 9 ½ 10 ½ 10 11 12 ½ 19 ½ 14 ½ 21 ½ 14 ½ 24
  1. Table 2 — Courses Given by Faculty, 1909-10 to 1955-56, by Rank*
    (Refers to Nearest Decimal point)
  1909-10 1919-20 1929-30 1939-40 1949-50 1955-56
Rank U G U G U G U G U G U G
Full Prof. 76 66 32 67 45 64 58 86 55 73 35 75
Assoc. Prof. 30 27 26 9 7 14 21 10
Asst. Prof. 14 36 24 10 4 17 27 8
Instructor & Lecturer 10 34 32 9 15 9 12 5 21 13 17 7
Total 100 100 100 100 100 100 100 100 100 100 100 100

* U = “undergraduate” and “undergraduate and graduate”;  G = “graduate”.
Source: Compiled from Course of Study Volumes.

  1. Table 3 — Percentage of Courses, Undergraduate and Graduate
Total No. of Courses % of Total Courses
(Exclusive of Bracketed Courses)
“Undergraduate” and
“Undergraduate & Graduate”
Graduate
(Inclusive of G.S.P.A. Economics Courses)
1909-10 12 88 12
1929-30 21 56 44
1939-40 32 39 61
1949-50 36 41 59
1955-56 38½ 38 62

From 1909 to 1929-30 the percentage of graduate courses was up from 12 to 44 per cent; but since 1929-30 the rise has been less spectacular. In Table 2, we note the courses, both undergraduate and graduate, given by men of various rank, from 1909-10 to 1955-56. The following points should be noted.

  1. The Increased Importance of Graduate Instruction. In 1909-10 there were but 1½ out of 12 courses, or 12 per cent, graduate courses. By 1929-30 courses were roughly evenly divided between graduate and undergraduate. By 1939-40 and 1949-50 the ratio was about 60 per cent graduate courses; and by 1955-56, 62 per cent of all courses were graduate courses, or 5 times as much relatively as in 1909-10.
  2. Reduced Undergraduate Instruction by Higher Ranking Members Faculty. Whereas in 1909-10 full professors accounted for 76 per cent of undergraduate course work, by 1955-56 they gave only 35 per cent of these courses; and there has been a marked decline since 1949-50. The total of undergraduate courses taught by them dropped from 1949-50 to 1955-56 by 3, or 37 per cent, and of graduate courses rose by 2¾ or 18 per cent. A similar trend is evident for associate professors, though from 1949-50 to 1955-56, the percentage of undergraduate courses taught by associate professors rose. It is a striking fact that in 1955-56, full professors taught 37 per cent less undergraduate courses and 1700 per cent more graduate courses than in 1909-10. In the former year there were 4 full professors, each responsible on the average for 2 full undergraduate courses and ¼ graduate courses. In 1955-56, 13 full professors averaged 1/3 of 1 undergraduate course and 1.4 graduate courses. (All 13 were not on full time). It is clear that the trend is away from undergraduate teaching for permanent members of the Department.
  3. Ibid., Statistical Summary. As might be expected, the percentage of all graduate courses taught by full professors tends to rise and of undergraduate courses to fall — the latter courses taught by professors declined from 76 per cent in 1909-10 to 45 per cent in 1929-30, and to 35 per cent by 1955-56.
  4. Number of Faculty by Rank. In this connection, the number at different ranks is of some interest. The full professors account for a somewhat larger proportion (teaching fellows omitted) than 50 years ago; but permanent appointments are an increased percentage.
  1909-10 1929-30 1939-40 1949-50 1955-56
Professors 4 5 12 13 13
Assoc. Professors 3 3 2 4
Asst. Professors 1 2 1 4 4
Lecturers and Instructors 3 2 3 4 3
Visiting, etc. Professors 2
(part-time)
3
(part-time)
1
Total (excl. Visiting) 8 12 19 23 24
———— ———— ———— ———— ———— ————
% Full Prof. (excl. Visiting) 50 42 63 57 54
% Permanent (incl. Permanent Lecturers) 50 67 89 74 75

Relations with the Graduate School of Public Administration

  1. Teaching Responsibilities of Economics Department in G.S.P.A. Our relations with the G.S.P.A. are of great importance. It is now close to 20 years since the G.S.P.A. was founded and yet the Department of Economics has never taken a long look at our relations. The Economics Department accounts for about one half the teaching burden of the G.S.P.A. and students in the G.S.P.A. account for about one third of all the graduate students in economics (on a full-time basis).
  2. Contributions of G.S.P.A to Economics Department. The G.S.P.A. has made an important contribution towards the Economics Department. It provides some research and secretarial help, good physical facilities, useful library, central facilities for students and faculty, an opportunity to give our students excellent seminars, and to meet outstanding scholars and practical men in government.
  3. Over-all Consideration of Number of G.S.P.A. Seminars. It may be that a decision should be made concerning the number of seminars. We tend to add one at a time, and the numbers now are at such a level that we may be putting a disproportionate amount of energy into these seminars. At any rate, net additions should be considered with care, given our available manpower. At present only 6 of the 18 permanent members of our faculty are not associated with the G.S.P.A.; and of the 6, Professors Dorfman and Duesenberry are about to participate. Of 27 courses to be given by permanent members of the Department, 7¼ will be as seminars in the G.S.P.A.

Library Problems

  1. Library Problems. Professor Arthur Cole retires this year. He has for many years been responsible for the acquisition of books in economics. Unless this responsibility is assumed by another, our economic collection will deteriorate. So far we have not been able to work out an arrangement acceptable to the Dean and the Director of the library. In my opinion, there is need for a central responsibility for library acquisitions in economics.

Fellowships

  1. Inadequate Fellowships. One of our most serious problems is fellowships. A study of fellowship funds announced as available to students suggested that Harvard was falling way behind. In a recent period of 5 years, five institution which are our strongest competitors had 30, 23, 20, 10, and 5 times as much money available for fellowships per Ph.D. granted in these five years. Increasingly we are losing the best students to rival institutions.
  2. Campaign for Additional Money. We have discussed this problem with Dean Bundy and Dean Elder, and also with our Visiting Committee. We have set up a committee consisting of Dean Mason, Professors Slichter, Dunlop and Harris to seek aggressively more fellowship funds. We are seeking these funds in the expectation that the major part of new funds will be available as additional funds for the Economics Department. Our goal is 6 fellowships at $2500 per year, or $15,000 per year additional. We discovered last year that by offering large fellowships to a limited number, we were more successful than in the past in attracting the more able candidates.
  3. Outside Fellowships. Our fellowship problem is eased by the availability of fellowships given by outside groups — governments, foundations etc. For example, Harvard received 5 of the 15 Wilson National fellowships for 1956-57. But it should be observed that there is often pressure to deny applicants access to the major universities and especially to Harvard. There is pressure to distribute widely, Moreover, a large proportion of these fellowship holders are often below our usual fellowship standards.

Research and Personnel Problems

  1. Competition of Research Fellowship Money for Potential Teachers. It is becoming increasingly easy for graduate students writing theses to receive fellowships that generally pay at least as much as a teaching fellowship. This year we lost 10 potential teachers as a result of these lucrative fellowships.
  2. Research Projects. Many of the Senior members of the staff are associated with large research projects, some of them of great significance. At least 9 of these projects may be classified as giant projects, three of them involving outlays of one half million or more dollars in the next 3-5 years. In 1955-56, Professor Leontief received almost one half million dollars to continue the projects of the Harvard Economic Group, and Dean Mason received $450,000 for a study of the New York Metropolitan area.
  3. Financing of Pay of Directors of Projects. It has always seemed to the Chairman, at least, that the foundations ought to pay part of the salary of the faculty members who direct these projects. When these projects are the major interest of those responsible for them, a case could be made for the foundation paying part of the salary of the relevant members of the faculty.
  4. Small Research Grants. It would be helpful to get some help from the Ford Foundation for small research projects especially for those who do not participate in the giant projects. I have had some preliminary discussion with the Ford Foundation, and I believe they would look with favor on an application for $25,000-30,000 per year for research help. Grants might vary from a few hundred dollars to $1,000-2,000 and be tied with specific projects. The great danger here is abuse of the privileges. Hence any such grant would have to be carefully administered – with some representation of outside economists on the committee.
  5. Secretarial Help. A related problem is that of secretarial help. Most of the Senior members, through administrative posts, control of seminars, editorial work, and research grants, manage to get the minimum amount of secretarial help. But 5 of our permanent members have virtually no access to secretaries and this is also true of most of our assistant professors. It would be helpful if some provision could be made for secretarial help for those without it. We realize this raises serious problems of finance.
  6. Personnel Changes. Professor Hansen retires this year and Professor Williams next year. We thus lose the best combination in money, cycles, and fiscal policy available anywhere. It is going to be difficult to fill this gap. Professor Black’s departure has also left a serious gap. We have added 2 very able assistant professors, Drs. J. Henderson and Valavanis, aside from two appointments (Drs. Moses and Conrad) in which the Economics Department shares one quarter of the cost. For 1957-58 and 1958-59, the Economics Department will have the services of Dr. E. Hoover for 3/7 of his time. We probably have the most able group of assistant professors in our history. It is not going to be easy to fill the gaps noted above, and make the most effective use of the young talent now in the Department. The Visiting Committee is again raising the question of a Professor of Business Enterprise, a matter to which we should give earnest attention. President Conant and Provost Buck were apparently prepared at the last discussion of this problem to provide an additional appointment for this purpose.
  7. Honors, etc. Dean Mason received an honorary degree from Harvard, and was a United States Representative at the United Nations Conference in Geneva on Peaceful Use of Atomic Energy.

Professor Hansen gave the Walgreen lectures at the University of Chicago.

Professor Harris served as Chairman of the Nor England Governors” Textile Committee,

Professor Galbraith advised the Indian Government on their Five Year Plan.

Professor Smithies was a Visiting Professor at Oxford and Professor

Kaysen at the London School of Economics.

 

Books:

Galbraith and Holton: Marketing Efficiency in Puerto Rico.

Harris: Keynes: Economist and Policy Maker.

Harris: New England Textiles and the New England Economy: Report to the Conference of New England Governors.

Kaysen: United States v. United Shoe Machinery Corporation: An Economic Analysis of an Anti-Trust Case.

Kaysen and Harris were two of the four co-authors of the American Business Creed.

 

Source: Harvard University Archives. Department of Economics, Correspondence and Papers (UAV 349.11). Box 2,  Folder: “Departmental Annual Reports to the Dean, 1955-”.

Image Source: Seymour E. Harris in The Harvard Class Album 1957.

 

Categories
Economics Programs Harvard Undergraduate

Harvard. Economics Chair annual reports to Dean, 1932-1941

 

This post takes us from the trough of the Great Depression to the eve of the U.S. entry into the Second World War. The items below are transcriptions of copies of reports written by the Harvard economics department chairmen of the time (Harold Hitchings Burbank (a.k.a. Burbie to his Buds) and Edward Hastings Chamberlin. Some chest-thumping, some whining, no notes of irony and definitely no flashes of wit…we all know this art form. Nevertheless some raw intelligence of value for working historians of economics of the present and future.

____________________________

November 12, 1932

Dear Dean Murdock,

Under the Faculty vote of December, 1931, the Chairman of each Department is requested to report in each half year to the Dean of the Faculty on the working of the plan recommended by the Committee on Instruction concerning Hour Examinations and Other Course Requirements. My report for the Department of Economics follows.

Acting on the Report from the Committee on Instruction, the Department of Economics on January 12, 1932 voted to observe the recommendations of the Committee. Following the Department meeting, I reported to you to the effect that the requirements of the Department of Economics were substantially in accord with the principles laid down by the Committee on Instruction. Ordinarily, we require not more than one Hour Examination in any one half year; ordinarily, we require not more than one thesis or report in any one half year. It is the standing rule of the Department of Economics and of the Division of History, Government, and Economics, that Senior candidates for Honors, who are writing Honors theses, shall be excused from the writing of any theses in courses within the Division. After a long discussion and with considerable reluctance, the Department voted that for Seniors who are candidates for Honors in the Division, Hour Examinations in courses within the Department shall be optional.

The vote of the Department was made known immediately to the students and observed in all of our undergraduate course (not of an introductory nature) during the second half of last year, and it is being observed in the current half year.

In the Division of History, Government, and Economics, we have had for many years a rule that all Seniors in good standing shall be exempted from final examinations in courses within the Division in their last half year. The result has been, of course, that after the April Hour Examinations, Seniors have paid little attention to courses within in the Division, and their attendance has been hardly more than occasional. The members of the Department who are more interested in courses than in General Examinations, and who perhaps doubt the efficacy of General Examinations, view this situation with increasing criticism.

When the Department voted the making of Hour Examinations optional for Seniors who are candidates for Honors, the doubting members were highly critical, fearing that our courses elected largely by Seniors would be entirely disrupted. From all that I can learn, I cannot see that there have been any untoward or undesirable results. In most of our “Senior” courses, the attendance until the Easter recess was satisfactory. Honors candidates attended lectures and, I believe, completed most of the required readings. Their records on the General Examinations were excellent. The Honors theses were among the best we have ever had.

A number of members of my Department and not a few members of the Departments of History and Government are strongly opposed to the new order. They make the point that we have in substance permitted an additional reduction in courses, that Senior Honor candidates are simply required to register in courses, but they have nether to attend them nor to do the work. All of these allegations are true enough, but it seems to me they are beside the point. To the extent that we have confidence in our examiners and tutors, I do not believe that in effect the requirements regarding the quality and quantity or work have been reduced.

The Department of History has recommended to the other departments of the Division the consideration of a motion which would require all senior candidates for Honors to complete whatever courses in History they elect. I think that probably the departments of the Division will consider in full detail the questions this motion involves.

Sincerely yours,
H. H. Burbank

Dean Kenneth B. Murdock
20 University Hall

Source: Harvard University Archives. Department of Economics, Correspondence and Papers (UAV 349.11). Box 2, Folder “Report to the Dean on the Department 1932-…”

____________________________

1933
[not found]

A copy of the report is not found with the others included in this post: Harvard University Archives. Department of Economics, Correspondence and Papers (UAV 349.11). Box 2, Folder “Report to the Dean on the Department 1932-…”

____________________________

October 15, 1934

Dear Dean Murdock,

I beg to submit the following report for the Department of Economics:

In this period of rapid economic evolution the problems presented to a group of university economists are both stimulating and perplexing. The changing pattern of our social and economic structure offers new data for analysis and at the same time calls for a testing of principle that involves new fields for both teaching and research.

There have been few periods in modern history more difficult to interpret, yet the responsibility for interpretation seems foremost among the duties devolving upon educational institutions. For many years the keystone of the introductory course in economics has been that the community has the right to expect political and economic leadership from the graduates of its colleges. Our undergraduate courses are directed toward the attainment of this end. But the teaching of political economy is an art not easily mastered even by those who give abundant evidence of intellectual leadership. In the instruction of undergraduates and in the training of teachers and scholars in our graduate school, the difficulties inherent in our subject must not be overlooked. The presentation of the data of economics makes demands upon the staff not felt in many other departments of the University. Looking toward the strengthening of our undergraduate instruction, the Department is now associating a number of the junior members of the staff with the senior members who are now in charge of the large lecture courses. In Money and Banking, in the Relations of Government to Industry, and in Public Finance, this experiment is advanced sufficiently to indicate its desirability.

At the same time that our teaching problems have become intensified the need for the results of research is pressing. In periods of accelerated social evolution involving political and economic experimentation, the demand for accurate data is insistent. Relatively, economics is a young science. The foundations of fact are still being established. Investigations that may have an important bearing upon government policy should not be delayed. The economists of this University have contributed largely to their subject, but always with scant facilities in material equipment and in time.

Among the many problems confronting us as a group, that of securing the time necessary for research is perhaps the most troublesome. To our exacting teaching requirements must be added the demands for public service. Since the establishment of this Department, the requests for such service heave been continuous. Of late the increasing calls have raised a question which must be considered by the University administration. The opportunities for service to governments are gratifying. Undoubtedly these services belong among the necessary functions of a university. But obviously they do divert a considerable part of our time and energy from our strictly defined duties. Over the years the University is enriched by such services, but at any given time the responsibilities attaching to teaching and research are interrupted. If the University Includes public service among its important functions, the personnel of the staffs affected should be so adjusted that the work can be performed without overtaxing our internal activities.

During the past your, the leave of absence of Professor John M. Williams was continued to allow him to serve as Economist of the Federal Reserve Bank of New York to advise on monetary and credit policies, and to direct research. In the latter part of the year, Professor Williams was called by the Department of State to investigate certain conditions in Brazil, Uraguay [sic], Argentina, and Chili [sic]  and to formulate policies of exchange controls. Daring the second half-year, Assistant Professor Edward H. Chamberlin was granted leave of absence to work with the Committee on Government Statistics and Information Services in Washington. Also, during the second half-year, though leave was not requested, Assistant Professor William T. Ham was in Washington frequently, serving as a member of the staff of the Labor Advisory Board of the National Recovery Administration. And also, though no leave was requested, Professor John D. Black devoted a substantial part of the year to public service. He served on a number of committees connected with the Agricultural Adjustment Administration and land utilization. At the request of Secretary Wallace, he organized and directed the activities of committees outlining programs of economic research in (1) the marketing of farm products and (2) farm population and rural life. Also at the request of the Secretary of Agriculture, he served with two others to coordinate the work of the Agricultural Adjustment Administration and the Bureau of Agricultural Economics of the United States Department of Agriculture. In the summer months, Drs. Alan Sweezy and Lauchlin B. Currie were called to the Treasury Department to serve as special investigators.

Owing to his illness, Professor Emeritus William Z. Ripley was unable to fulfill his duties as President of the American Economic Association. In his absence, Professor Abbott P. Usher, first Vice-President of the Association, was in charge of the December, 1933 session.

Notable among our publications of the year were Twenty Years of Federal Reserve Policy, by S. E. Harris, and The Theory of Monopolistic Competition, by E. H. Chamberlin. Because of its significance for immediate practical application, I am including at this point the Report of the Committee on Model State and Local Taxation, by Professor C. J. Bullock’s committee of the National Tax Association. Also at this point, mention should be made of Economics of the Recovery Program, by seven members of the Department. In the course of the year, about forty-five articles were contributed to scientific journals by various members of the Department.

Within the limitations described above, the research work of the staff is going forward at a satisfactory rate. Investigations in the following subjects are well advanced: History of the Industrial Revolution; Development of Banking and Credit in the Sixteenth and Seventeenth Centuries; Evolution of English Company Law; Economic Fluctuations; Nature and Effects of Inflation; Index Numbers; Municipal Ownership of Public Utilities; State and Local Taxation; Unbalanced Budgets; The National Income; New England Agriculture; The Economics of Agricultural Production; German Trade Unionism; The Fundamentals of Sociology; Economics and Politics; Socialism as an International Movement.

A considerable number of these projects are nearing completion and should be ready for publication shortly. A large project on the relation of Government to Industry involving the efforts of a number of the staff is in its initial stages. This subject is of such immediate importance that other plans for research are being put aside until it can be carried to its completion. The Quarterly Journal of Economies has continued its usual high standard. During the year, five substantial volumes were added to the Harvard Economic Studies.

Again I would press the point that the potential research capacity of the Department is severely handicapped by the demands of teaching and public service.

Sincerely yours,
H. H. Burbank

Dean Kenneth B. Murdock
20 University Hall

Source: Harvard University Archives. Department of Economics, Correspondence and Papers (UAV 349.11). Box 2, Folder “Report to the Dean on the Department 1932-…”

____________________________

October 18, 1935

Dear Dean Birkhoff:

I beg to submit the following report for the Department of Economics.

In the report of last year the effects of the contemporary political and economic situation upon our problems of teaching and research were discussed briefly. More than ever we are aware of the responsibilities incumbent upon the teacher of Economics in this period of rapid and far-reaching change. Our undergraduate instruction had been, and is, receiving particular attention. A few years ago we began experimentally the association of a number of the junior members of the staff with the senior members who are nominally in charge of the larger lecture courses. We are quite convinced that this method of instruction is most effective. Also there is a positive, although perhaps incidental, advantage in this arrangement in that it relieves the pressure for the multiplication of undergraduate courses.

I find it necessary to stress again the problem presented by the demands upon our staff for services to the public. We believe that public service belongs among the necessary functions of a university. But under existing conditions large demands for public service at any given time bring serious interruptions to both research and instruction. “If the University includes public service among its important functions the personnel of the staffs affected should be so adjusted that the additional work can be performed without taxing severely our internal activities.”

I am very happy, to write that Professor Chamberlin’s “The Theory of Monopolistic Competition”, published somewhat over a year ago, has won immediate recognition as a foremost contribution to economic theory. During the past year two books of unusual importance have appeared,—Professor John D. Black, “The Dairy Industry and the A.A.A.”, and Professor Sumner Slichter, “Towards Stability”. Six manuscripts have been completed, and should appear in book form during the present year. It is significant that five of these books have been written by the younger members of our Department whose teaching duties have been mainly of a tutorial nature. Among the publications I should note the report submitted to the Treasury Department on the “Objectives and Criteria of Monetary Policy” by Dr. Alan Sweezy, and the report to the State Department on “Foreign Exchange Control in Latin America” by Professor John Williams.

In addition to the above volumes and reports the members of the Department published somewhat over fifty articles in the scientific journals of our subject. Some of these contributions are of major importance.

The investigations of the staff are being carried forward as satisfactorily as possible with the limited facilities that are at our disposal. Two researches on a very large scale have to do with the general subject of the Trade Cycle and the Relation of Government to Industry. Numerous important, but less extensive, investigations are in process.

Perhaps I should note here that a generous grant from the Rockefeller Foundation enabled the Department to undertake the continuation of the Review of Economic Statistics and the fundamental research that is involved in this publication, The Quarterly Journal of Economics long published by the members of this Department, together with the Review of Economic Statistics, are among the more important activities of the Department. In the course of the year three volumes more added to the Harvard Economic Studies.

As in my last report, I would again bring to your attention the disturbing fact that the potential research capacity of the Department is handicapped severely by the demands of administration, teaching, and public service.

Very sincerely yours,
H. H. Burbank

Dean George D. Birkhoff

Source: Harvard University Archives. Department of Economics, Correspondence and Papers (UAV 349.11). Box 2, Folder “Report to the Dean on the Department 1932-…”

____________________________

October 15, 1936

Dear Dean Birkhoff:

I beg to submit the following report for the Department of Economics.

I find it necessary to emphasize again the effects of the contemporary political and economic situation upon our problems of teaching and research. It had been necessary to bring these matters to your attention in both of the preceding years, since they present such important problems to us. We feel an increasingly positive responsibility regarding out undergraduate instruction in this period of rapid and far-reaching change.

We have continued the experiment begun some few years ago of the association of a number of the junior members of the staff with the senior members who are in charge of the large lecture courses. We believe that we are improving our instruction by this method, and at the same time this arrangement tends to relieve the pressure for the multiplication of undergraduate courses.

Perhaps as a result of the general social situation the elections of our undergraduate courses and the number of concentrators in Economics have increased very heavily. The problems of instruction presented by these overwhelming numbers are intensified perhaps by the personnel situation in which the Department finds itself. During the last dozen years the personnel of this Department—one of the largest in the University—has been changed completely. For a quarter of a century a group of eminent economists brought great prestige to the University. With the resignation of Professor Gay the active services of this group has come to an end. One cannot speak of replacing these scholars. They were unique both as individuals and as a group. Their leadership and their scholarship has left a lasting impression on the development of Economics. In the course of the passing of this group a now Department has been brought together. This new and younger Department is assuming full responsibility at the very time when questions of teaching and new methods of research are becoming insistent.

The demands upon members of our staff for public service continue. It has seemed expedient to encourage some few members to give their time and energy for public purposes. But with a minimum teaching force it has not been possible for all members of the Department to comply with the requests made. The public service relations of faculty members remains a question for the University to consider.

The Quarterly Journal of Economics celebrates this year its fiftieth anniversary. For forty years this Journal has won and held its prestige under the editorship of Professor F. W. Taussig. Professor Taussig, now emeritus, has graciously consented to continue as editor during the present year, but very shortly it will be necessary for us to provide for the editorial direction of this very important publication.

In an earlier report to you I indicated the activities of the Department in connection with the Review of Economic Statistics. The scientific work underlying this publication, as well as the journal itself, is now under the direction of a committee of the Department. The Review continues as a vehicle of publication of the results of investigations here and elsewhere regarding the business cycle. We have ambitious plans for the Review, and we have every reason to believe that its scientific usefulness will increase.

There is little question that, the research activities of practically all members of the staff have been curtailed by the heavy teaching loads which have been imposed. However, the research programs of various members and of various groups within the Department have shown marked progress in the past year. As I have indicated in an earlier report the research activities of our members are of two somewhat different types. Numerous members of the staff working altogether independently are pursuing their own researches while others working as a group are developing particular aspects of a well devised project in research. In the social sciences this latter type of work is rapidly assuming importance. In general it is this type of research which receives the support of the large foundations. Within our own group there are a number of projects of this character. Messrs. Mason, Chamberlin, Wallace, Cassels, Reynolds, and Alan Sweezy are developing Industrial Organization and Control. In the process of the exploration of this subject numerous independent volumes and studies will appear. Professors Mason, Chamberlin and Dr. Wallace are already well advanced in their study of monopolistic combinations and expect to complete it in about one year. Professor Cassels and Dr. Reynolds expect to finish their study on Canadian combinations this year, and Dr. Alan Sweezy is at work on investment policies. Dr. Wallace’s monograph, Market Control in the Aluminum Industry, is now going to press, and Dr. Abbott’s monograph on The Rise of the Business Corporation has just appeared and is being, used by our undergraduate courses. The full development of this program will take a number of years, but its completion will mark, I believe, a very significant chapter in research in the relation of government to industry.

Another cooperative project on the Farm Credit Administration is being carried on by Professors Black and Harris and Dr. Galbraith, largely with the assistance of grants from the Committee on Research in the Social Sciences. Professor Black is working on the cooperative aspects of the Farm Credit Administration’s policies. Professor Harris is working on the monetary and recovery aspects of the Farm Credit Administration’s loan operations. Dr. Galbraith is working on the structural aspects of the Farm Credit Administration and the mortgage, credit and production loan policies. Numerous articles resulting from this research have been published in scientific periodicals.

Professors Crum, Wilson, and Black are conducting a study of the relation of weather and other natural phenomena with the economic cycle. This study is partly financed by the United States Department of Agriculture.

I believe I have mentioned to you and to President Conant in conversation the plans which are being developed for large research projects in collaboration with the National Bureau of Economic Research.

In addition to these cooperative projects all members of the Department are pursuing work along the lines of their individual interests. Professor Schumpeter’s study of time series and cyclical fluctuations is practically completed, and he hopes to send it to press by December. Professor Haberler’s major contribution—The Theory of International Trade and Its Application to Commercial Policy has been translated and is now available in English. For the past two years Professor Haberler has been working at Geneva on the Nature and Causes of the Recurrence of Economic Depressions which is soon to be published by the League of Nations. We are hoping to provide facilities for him so that the important research may be continued at Harvard. Professor Frickey’s study on a Survey of Time Series Analysis and Its Relation to Economic Theory is well advanced. The statistical work on the first volume has been completed, and he hopes to have it written by the middle of this present academic year. The statistical work on the second volume has been completed in part. Already two significant articles have been published. Professor Cole’s recent study in Fluctuations in American Business, written in collaboration with Professor W. B. Smith, was published late in 1935. Dr. Oakes’ investigations in Massachusetts Town Finance, the winner of the Wells Prize for 1935-36, is now being printed. Professor Chamberlin has continued to elaborate his Theory of Monopolistic Competition which is winning wide recognition among economist the world over. Numerous articles, some sixty in number, from members of the staff have appeared in various scientific periodicals in the course of the year.

Very sincerely yours,
H. H. Burbank

Dean George D. Birkhoff
20 University Hall
Cambridge, Massachusetts

[Separate sheet following: I should have included Professor Harris’ Exchange Depreciation, Its Theory and History. We believe that this new book, which is being published today, will take Its place beside the significant contributions Professor Harris has made in the last half-dozen years, particularly his Monetary Problems of the British Empire and Twenty Years of Federal Reserve Policy.]

Source: Harvard University Archives. Department of Economics, Correspondence and Papers (UAV 349.11). Box 2, Folder “Report to the Dean on the Department 1932-…”

____________________________

October 21, 1937

Dear Dean Birkhoff:

I beg to submit the following report for the Department of Economics.

Previous reports of the Department of Economics have brought to your attention the effect of the political and economic situation upon our problems of teaching and research. It is still necessary to point out that the positive responsibility of the Department regarding undergraduate instruction has not lessened.

The election of our undergraduate courses remains at substantially the high level of recent years, while the number of concentrators continues to increase.

Last year I mentioned that with the resignation of Professor Gay the active services of the senior members of this Department, had come to an end. At this point it seems necessary to put into writing a matter I have discussed with you in conversation which has important ramifications. Coincident with the resignation of Professor Gay there were increased elections in certain of our courses that involve a large degree of individual instruction and also on an increase in the number of students demanding tutorial supervision. To meet these latter problems it was necessary to add to our staff a group of young men to carry on the instruction in the elementary course, Accounting, Statistics, Money and Banking, and so on. With increased numbers in courses demanding increased instruction, increased cost cannot be avoided; but it seems to us that this increasing cost because of increasing should not result in less effective intellectual leadership. To transfer a considerable part of the salary released by a retiring professor of distinguished accomplishment to the support of routine instruction in middle group courses seems to us not to be wise University policy.

Professor Taussig has resigned as editor of the Quarterly Journal of Economies. For the time being, committee of the Department will undertake the editorial direction of this publication.

The Review of Economic Statistics, which appears under the direction of a committee of the Department, is financed by funds from the Rockefeller Foundation. Should the grant be continued, it is expected that the research activities of the committee will be increased.

Not less than ten members of the Department are concerned with the activities of the Graduate School of Public Administration. In some instances—as in the case of Dean Williams—their work in the School has been compensated by a reduction of work in the Department, but for the most part the activities in the new School are simply in addition to the duties of the staff members.

The Committee on Research in the Social Sciences, of which Professor Black is Chairman, is working in close cooperation with the National Bureau of Economic Research and its cooperating University agencies. Principle among them is the project upon Fiscal Policy for which Professor Crum is acting as Chairman.

The responsibilities and activities of members of the Department tend in some instances to change the direction of our research, but in only too many instances they also tend to retard our research.

In all directions, however, the research activities of the members of the Department were sustained, with six books and approximately sixty articles appearing. Special mention should he made of the following books:

Three Years of the AAA by John D. Black

A Study of Fluid Milk Prices by John M. Cassels. Wells Prize Essay of 1934-35

Professor Chamberlin’s significant volume, The Theory of Monopolistic Competition has been revised.

Prosperity and Depression by Gottfried Haberler

Exchange Depreciation by S. E. Harris. (Came from the press last fall, and mentioned a year ago.)

Studies in Massachusetts Town Finance by E. E. Oakes. Wells Prize Essay of 1935-36

Professor Schumpeter’s book on Business Cycles has been completed, and is now ready for the press.

Economic History of Europe since 1750 by Usher, Bowden, and Karpovich

Explorations in Economics. Essays in Honor of F. W. Taussig contains contributions by most of the members of the staff.

Very sincerely yours,
H. H. Burbank

Dean George D. Birkhoff
20 University Hall
Cambridge, Massachusetts

Source: Harvard University Archives. Department of Economics, Correspondence and Papers (UAV 349.11). Box 2, Folder “Report to the Dean on the Department 1932-…”

____________________________

October 15, 1938

Dear Dean Birkhoff,

I beg to submit the following report for the Department of Economics.

As in previous years I am very happy, to be able to record that the research activities of the officers of the Department have been sustained. In the last two years I have been, able to enumerate an unusually large number of books actually published together with numerous contributions to our periodical literature. In the present year the number of volumes is smaller since the research activities of our staff are still in process. The most notable volumes are Professor Hansen’s Full Recovery or Stagnation and Professor Wallace’s Market Control in the Aluminum Industry. Professor Haberler devoted the major part of the year, and spent the summer abroad, revising his Prosperity and Depression. Also the volume by Professor Crum and Associates on Economic Statistics has been revised.

In all, some fifty or sixty periodical contributions have been made by members of the staff. Notable among these contributions have been the articles by Professor Slichter on “The Downturn of 1937” in the Review of Economic Statistics for August, 1938.

It fell to the lot of the officers of this Department, together with the officers of the Department of Government, to develop instruction in the Littauer School of Public Administration during the past year. Without going into the details of the principles upon which this instruction is based, it may be noted that research courses of a very advanced nature constitute the core of the work of the School. Professors Williams, Hansen, Black, Mason, Slichter, and Wallace are devoting a considerable proportion of their time to this work. It is expected and hoped that these activities will result in an increase in our contributions.

The grant of funds from the Rockefeller Foundation to subsidize the research underlying the Review of Economic Statistics expired with the closing of the fiscal year. This contribution made it possible to continue the Review, and to maintain the scholarly level of the contributions. In the course of the year the Review published a number of the contributions of the staff. Other contributions are nearing completion, and will be published in the present year. The accomplishments or Professors Crum and Haberler as Managing Editors of the Review should be noted. They have succeeded in restoring the very high level of scholarship which characterized the Review a decade ago. We believe that the Review in its present form adds materially to the prestige of the Department and the University.

Also I am happy to note that the Quarterly Journal of Economics under its new editorial staff is maintaining its high position.

There is little to be added to the points which have been discussed in previous reports. The Department finds itself fully occupied with the continuation of its traditional activities and the assumption of such new duties as are involved in the Graduate School of Public Administration. If the personnel of the Department remains constant, it will be necessary to reduce our activities, either in research, in teaching, or in both.

Last fall at a dinner of the Committee to Visit the Department of Economics I reported in some detail regarding the increasing activities of members of the Department. This report led to the appointment of a committee to investigate the budgetary situation of the Department. The investigation conducted under the direction of Mr. George May of Price, Waterhouse, made some very interesting disclosures regarding the increasing load of the Department.

I believe that problems of undergraduate and graduate instruction, the tutorial situation, and the public service contributions of our members have been discussed sufficiently in previous reports. I can only repeat that “there is little question that the research activities of practically all members of the staff have been curtailed by the heavy loads of teaching and administration.

Very sincerely yours,
H. H. Burbank

Dean George D. Birkhoff
20 University Hall
Cambridge, Massachusetts

 

Source: Harvard University Archives. Department of Economics, Correspondence and Papers (UAV 349.11). Box 2, Folder “Report to the Dean on the Department 1932-…”

____________________________

October 16, 1939

Dear Dean Ferguson:

In accord with your recent request, I submit herewith a report of the work by the Department of Economies for the past year.

Honors have been bestowed upon members of the Department as follows: Professor Schumpeter has received an honorary Ph.D. from the University of Sofia, Bulgaria, and Professor Leontief has been elected a Fellow of the Econometric Society. Professor Williams was elected a Vice-President of the American Economic Association.

In the field of publications, the outstanding event is the final appearance of Professor Schumpeter’s two volume work on Business Cycles: A Theoretical, Historical and Statistical Analysis of the Capitalistic Process. The fruition of years of study and research, this book is of especial interest as the first major work of Professor Schumpeter in the English language, his well-known Theory of Economic Development having appeared first in German before its translation into English much later. Other books actually appearing within the academic year (the fall of 1938) were referred to in our last report, such as Professor Hansen’s Full Recovery or Stagnation?, a revision of the volume on Economic Statistics by Professor Crum and associates, and a new, enlarged and revised edition of Prosperity and Depression by Professor Haberler (published by the League of Nations). During the year arrangements have been completed for the translation into Japanese of A History of Mechanical Inventions by Professor Usher. For some years Professor Emeritus F. W. Taussig has been at work on a thorough-going revision of his textbook on the Principles of Economics. Volume I appeared last spring, Volume 2 is in the press and will appear very shortly. This much needed revision (the last was in 1921) may regain for Professor Taussig’s text some of the preeminence it held in an earlier period before it had become so badly out of date. Politics, Finance and Consequences by Professor Emeritus C. J. Bullock, the result of continuing research since his retirement, has been published during the past year in the Harvard Economic Studies. A book of which Mr. Paul M. Sweezy was a prominent co-author, An Economic Program for American Democracy, is popularly supposed to have been influential in putting the stamp of economic authority upon recent economic policies of the Federal Government. Finally, some sixty-odd articles, addresses, and reviews by members of the Department have appeared in journals, both professional and popular, during the past year.

A matter not mentioned in our last report was a new policy adopted by the Quarterly Journal of Economics of publishing at intervals of approximately one year a series of supplements devoted to articles and studies of interest to scholars but of such length as to make their publication in the regular issues impractical. These supplements are sent to subscribers without charge, and additional copies are sold separately. The first of these appeared in May 1938, Rudimentary Mathematics for Economists and Statisticians by Professor Crum. Two other manuscripts have been accepted and will appear shortly.

The Committee on Problems of the Business Cycle has carried on the publication of the quarterly Review of Economic Statistics but because of the expiration of its grant of research money many of its new research investigation have been greatly curtailed. Quarterly issues of the Review of Economic Statistics, in addition to carrying the studies of current economic history which present a quarterly record of economic statistics for the United States with their interpretation, have published a wide range of articles on various aspects of the trade cycle problem. Several of these articles have been contributed by foreign specialists but more than half were produced by American writers (in this connection we may note that about one-fourth of the subscribers are located abroad). In addition to the normal research activities involved in studying current history the Committee has financed during the year a continuation of the special investigation by Dr. J. B. Hubbard of the remarkable developments in the issuance of securities since 1933. A further article in Dr. Hubbard’s series will appear in the issue of November 1939.

Mention has been made in previous reports of the burden placed upon particular members of the Department and thus upon the group as a whole by the responsibilities of public service. These responsibilities have continued and expanded during the past year. The adjustment of this burden is a pressing problem. Its immediate influence upon both teaching and research is adverse, yet no ready solution appears at hand. The additional burden of uncompensated teaching in the Graduate School of Public Administration presents an even more serious problem. For the most part the seminars and other activities of this School constitute a net additional load for those members of the Department responsible for them, and inevitably throw a heavier burden of administrative and other work upon others not directly concerned. Budgetary allowance for courses given within the School is an obvious answer to this problem, whenever it may become possible.

You have asked, among other things. for an account of “any changes in the methods of instruction”, of the Department. The changes here have been revolutionary. Over a long period of years there has been built up in the Department a staff of trained instructors and tutors, carrying on established traditions of teaching and constantly experimenting in the adaptation of methods to new problems. These men were sifted constantly, and the best of them retained for a substantial period, after which, if not advanced, they were without exception placed to advantage elsewhere. In view of the singular success with which in the past the personnel problem has been handled in Economics, it is not surprising that the Department is unanimous in viewing with dismay and discouragement the situation in which we now find ourselves. Fifteen teachers and tutors at the instructor or assistant professor level have left us within the past year, seven the preceding year. The general effect upon teaching may be indicated by the tutorial situation. Sixty-seven per cent of the students concentrating in Economics this year are tutored by men of two years or less experience, forty-three per cent by men of no tutorial experience whatsoever, Furthermore, it has been our policy in the past to stagger new men as between tutoring and Economics A, having them start in with either one alone and take up the other the following year. This fall we have been obliged to take on five men who are both teaching Economics A and tutoring for the first time. It has been our policy also to provide more experienced instruction in middle group courses through a period of apprenticeship in Economics A. This fall we have been obliged to put men of no classroom experience whatever directly into middle group courses. We are already experiencing in acute form the devastating effects upon instruction of a rapid turnover, brought on by the mass exodus of last year.

It takes time (and patience on the part of someone) to train men in the discussion method of teaching Economics which has been developed with such success in Economics A at Harvard University. Much is learned by slow experience, by making mistakes and by discussing techniques with fellow instructors, especially with those who have been through the mill. It is impossible to assimilate new men unless the collective experience of the group is maintained at a fairly high level. Furthermore, it seems unlikely that anyone in the Department will be interested in training them unless a substantial portion stay long enough to make it worth while.

Very sincerely yours,
H. H. Chamberlin

Dean W. S. Ferguson
20 University Hall
Cambridge, Massachusetts

Source: Harvard University Archives. Department of Economics, Correspondence and Papers (UAV 349.11). Box 2, Folder “Report to the Dean on the Department 1932-…”

____________________________

October 15, 1940

Dear Dean Ferguson:

I submit herewith a report of the work by the Department of Economics for the past year. There is very little to report—no events or changes of outstanding importance, and only a few isolated items which might be of interest.

Professor Black has been elected to honorary membership in the Swedish Royal Society of Agriculture. Professor Slichter has been honored by appointment as Lamont University Professor.

In the field of publications there is the usual long list of articles in the professional periodicals, but no major work of importance by any member of the Department. Professor Usher’s History of Mechanical Inventions was during the year translated into Japanese. Also in the field of publications it is of interest that there has been begun under the supervision of a committee in the Department and financed in part by a grant from the A. W. Shaw Fund a new series entitled The Harvard Studies in Monopoly and Competition. The first two volumes of this series appeared within the year, — the first, Corporate Size and Earning Power, by Professor W. L. Crum, and the second, Control of Competition in Canada, by Lloyd Reynolds.

The Committee on Problems of the Business Cycle has continued publication of the quarterly Review of Economic Statistics. In place of the general reviews of current economic developments in the United States, which in earlier years had been regular features of each quarterly issue, the Review introduced this past year the policy of presenting each quarter an article pertaining to some specific problem of current interest. The November 1939 issue contained a study of the impact of the war on America commodity prices; the February 1940 number included a study of the current gold problem and the American economy; a review of recent developments in agriculture and the influences of the war on American agriculture appeared in May; while the August 1940 issue presented a comparison and evaluation of various estimates of unemployment in the United States. These studies have been made by members of the Department, with the Committee staff contributing assistance, whenever it was desired, in the preparation of the articles for publication. As in previous years, the Review has also presented articles covering a wide range of studies on various trade cycle problems; and the Review staff has continued the compilation of selected current economic series which have been used in research studies by Department members and graduate student within the Department.

There have been no important changes in policy in the year by the Quarterly Journal of Economics. The policy begun the previous year of publishing occasional supplements sent to subscribers without charge has been continued. Two supplements appeared during the year, Exchange Control in Austria and Hungary and Exchange Control in Germany, both by Professor Howard S. Ellis. Through an arrangement with the Harvard Economic Studies they will shortly appear in that series as a single volume.

During the year Professor Emeritus Frank W. Taussig attained his eightieth birthday. A tribute and greeting was presented to him on this occasion signed by some two hundred of his former students.

I call attention again to the continuing problem of the added burden to members of the Department for uncompensated teaching in the Graduate School of Public Administration. The situation here remains substantially as described in my last report. It remains one of the most serious problems which the Department has to meet in maintaining the standards of its instruction.

The quality of instruction given by the Department continues to suffer from the heavy losses in the junior personnel during the past few years. Sixty-four per cent of the students concentrating in Economics this year are tutored by men of two years or less experience, fifty-five per cent by men of one year or less. The difficulties of maintaining satisfactory instruction with such a rapid turnover remain almost insuperable, and concentration in Economics which has fallen off steadily over the past four years slumped most disastrously for the year 1940-41. Although most of the liquidation of our more experienced instructors and tutors had taken place before the year on which I am reporting, we have during that year again lost a number of our best men because of the limited inducement which could be offered for them to remain with us even for a short period.

Sincerely yours,
H. H. Chamberlin

Dean W. S. Ferguson
5 University Hall
Cambridge, Massachusetts

 

Source: Harvard University Archives. Department of Economics, Correspondence and Papers (UAV 349.11). Box 2, Folder “Report to the Dean on the Department 1932-…”

____________________________

October 15, 1941

Dear Dean Ferguson:

I submit herewith a report on the work of the Department of Economics covering the past year.

Professor Slichter has been elected President of the American Economic Association. This is the third time in the past five years that this honor has gone to an economist from Harvard, Professor Sprague having been elected in 1937-38 and Professor Hansen in 1938-39.

In the field of publications there have appeared, in addition to the usual long list of articles, several books of possible importance. I should mention especially Professor Slichter’s Union Policies and Industrial Management, Professor Leontief’s The Structure of American Economy: An Empirical Application of Equilibrium Analysis, and Dr. Triffin’s Monopolistic Competition and General Equilibrium Theory. The latter appeared in the Harvard Economic Studies of which there have now been published 70 volumes, four within the past year. The new series of Harvard Studies in Monopoly and Competition has been augmented by two new volumes during the past year, bringing the total to four. Professor Usher’s History of Mechanical Inventions has again been translated, this time into Spanish. During the past year an arrangement was made with the Rockefeller Foundation (for the current year only) which if continued may prove to be of real importance to the members of our Department. Professor Crum has been relieved of one-half of his teaching duties for research through the payment by the Foundation of the salary of someone to replace him in his teaching assignment. In addition to providing possibilities for research to members of the Department, such an arrangement would have the added advantage of making it possible to invite to Harvard for short period either possible candidates for permanent appointments or others whose presence here for one year would prove stimulating to our students.

Again I call attention to the problem of the added burden to members of the Department for uncompensated teaching in the Graduate School of Public Administration. This has been from the beginning a serious matter in maintaining standards of instruction. It is especially a factor in concentrating the activities of the older members of the Department in the graduate field, leaving undergraduate instruction to be taken care of in undue degree by younger men whose experience on the average seems to decline further each year.

The quality of instruction by the junior staff continues to be a grave concern to our Department. Last year I mentioned that 64 per cent of the students concentrating in Economics were tutored by men of two years or less experience. This year the percentage has increased to 72, and the problem of finding enough experienced and competent tutors in the right fields for distinction seniors has become impossible to solve. The general situation is reflected also in Economics A where the percentage of new instructors has jumped alarmingly for the current year. For the five years 1936-41 the sections taught by new men averaged 24 per cent of the total. For the current year 39 per cent of the sections are taught by new men. For the same five years the sections taught by men of one year or less experience averaged 45 per cent of the total. For the current year this figure has advanced to 61 per cent. The large volume of complaints on the part of students as to the inexperience of their tutors and Economics A section instructors leaves no doubt in the minds of the Department that the continuing decline in concentration in Economies is mainly a reflection of this situation. In view of the competing opportunities for our younger men which have repeatedly been pointed out the problem for our Department continues to be not to maintain a high rate of turnover as the present rules of tenure seem designed to do, but to be able through more flexible arrangements both with respect to tenure and to salaries to maintain a staff sufficiently experienced to give satisfactory instruction to our undergraduates. Such instruction is clearly not being given at the present time.

Sincerely yours,
H. H. Chamberlin

Dean W. S. Ferguson
5 University Hall
Cambridge, Massachusetts

Source: Harvard University Archives. Department of Economics, Correspondence and Papers (UAV 349.11). Box 2, Folder “Report to the Dean on the Department 1932-…”

Image Source: Harold Hitchings Burbank from the Harvard Class Album 1934.

 

Categories
Agricultural Economics Chicago Economists

Chicago. Economics Ph.D. alumnus, Edwin Ferdinand Dummeier, 1926

 

From the University of Chicago economics department records we can assemble a fairly complete account of the process of earning a doctorate in economics for the agricultural economist Edwin F. Dummeier who entered the Chicago program with a year’s worth of graduate credit. Dummeier’s five quarters in Chicago (from Summer 1925 through Summer 1926) in residence seems to be a lower bound at a time when the official regulations had been changed to state that as a general rule three years residence in graduate studies were expected of Ph.D. degree candidates. 

It appears to me that Dummeier’s undergraduate degree at L.S.U. was the result of regular summer school attendance while teaching/administering during the regular school year. His collection of graduate credits from the Universities of California, Wisconsin, and Colorado also show a considerable portion of summer school credit. It is interesting to see that he could apparently be appointed the principal of a Louisiana high school without having a completed college education. 

________________________

Brief c.v. of Edwin Ferdinand Dummeier

1887, April 4. Born in Metropolis, Illinois.

1910-1917. Principal of Leesville, Louisiana High School

1917-1918. Principal of Minden High School, Webster Parish, Louisiana.

1918. A.B. Louisiana State University

1921. M.A. University of Colorado.

1921-23. Instructor in economics, Washington State College (Pullman, WA).

1923-1925. Assistant Professor, Washington State College (Pullman, WA).

1926. Ph.D. University of Chicago. Thesis: The marketing of Pacific coast fruits in Chicago.

1926-46. Professor of Economics, State College of Washington, Pullman, Wash.

1944, June 19. Married Binna Mason, school teacher

1946, June 17. Died in Salt Lake City, Utah.

Biggest publication:

Edwin F Dummeier and Richard Brooks Heflebower. Economics: with applications to agriculture. New York: McGraw-Hill, 1940.

________________________

Dummeier’s application for graduate credit towards an economics Ph.D. from Chicago

The University of Chicago
The Graduate School of Arts and Literature
Office of the Dean

August 19, 1925

Mr. J. A. Field
Faculty Exchange:

I enclose application for graduate credit from Mr. Edwin F. Dummeier who is a graduate student in residence this quarter. While he is doing most of his work in Commerce and Administration at present, he wishes to go into Political Economy, and so I am asking you to estimate the amount of credit in Pol. Econ. that ought to be given in majors and in quarters for the work he lists. Please return the certificates from the University of California and the University of Wisconsin.

Sincerely yours,
[signed]
G. J. Laing
Dean

GJL:M

________________________

Department will recognize three quarters of graduate work

August 29, 1925

Dean G. J. Laing
University of Chicago
Faculty Exchange

My dear Mr. Laing:

I enclose herewith application for graduate credit for Edwin F. Dummeier which I have certified as representing in my judgment the substantial equivalent of three quarters of graduate work in Political Economy.

Sincerely yours,

[unsigned copy, J.A. Field]

JAF:MLH
Enclosure

________________________

Dummeier proposing his examination fields and requesting departmental review of all his coursework to identify any further course requirements

5757 University Avenue, Chicago, Illinois,
January 21, 1926

Professor L.C. Marshall, Chairman,
Department of Political Economy
The University of Chicago.

Dear Sir:

Announcements from the Department of Political Economy to persons intending to become candidates for the Ph.D. degree state that “the candidate, subject to the advice and approval of the Department,” may choose his fields for specialization and written examination from designated lists. Other announcements of the University state that in the Graduate Schools of Arts, Literature and Science the courses to be offered must be “approved by the Deans of the Graduate Schools at least six months before the degree is conferred. The individual courses must receive the approval of the heads of the departments concerned.” It is also stated that the Department of Political Economy will ordinarily approve as an essential part of a student’s preparation for the degree a considerable amount of work in allied departments.”

In consideration of these announcements I am herby submitting the following statement of fields which, with the approval of the Department, I propose to designate as fields of specialization and examination: (1) General Economic Theory; (2) Market Structures and Functions, this being the thesis field; (3) The Pecuniary and Financial System; (4) Transportation and Communication.

Furthermore, I am submitting a list of courses in the past pursued and a statement of courses which I have taught, in order that the Department may take definite action of a character which will enable me to plan my work in the future with an assurance that all course requirements are being met.

My undergraduate work included courses in the principles of economics and accounting. It also included courses in history and political science.

Graduate work thus far completed and courses for which I am registered for this quarter are as follows:

Political Economy

At the University of Colorado, six quarters, 1919-1921
Money and Banking 24 weeks 2 hours per week
Taxation 36 weeks 2 hours per week
Socialism 24 weeks 2 hours per week
Immigration 6 weeks 5 hours per week
Business Organization 6 weeks 5 hours per week
Seminar in Economics 12 weeks 2 hours per week
Thesis, “Financing Public Education in Colorado,” 6 quarter hours credit.

 

At the University of California, summer 1923
Transportation, principles [& Hist. (Dixon)] 6 weeks 5 hours per week
Transportation, current problems 6 weeks 5 hours per week
Pacific Coast Rate Problems 6 weeks 5 hours per week

 

At the University of Wisconsin, summer 1924
The Classical Economists [Physiocrats thru J. S. Mill] 6 weeks 5 hours per week
Farmer Movements 6 weeks 5 hours per week
Statistics 6 weeks 7½ hours per week

 

At the University of Chicago, summer, spring, and winter Qtrs. 1925-26
Course No.
334 Money and Prices 1 major
388A Cooperative Marketing 1 major
388B Marketing Farm Products 1 major
301 Neoclassical Economics 1 major
345 Personnel Administration 1 major
386 Terminal Marketing Research 1 major
C & A. 375 Business Forecasting 1 major
335 Bus.Finance and Investment 1 major
499 Terminal Marketing Research 1 major

 

Sociology

At the University of Colorado, 1919-1921
Social Problems (poverty) 12 weeks 2 hours per week
Rural Sociology 12 weeks 2 hours per week
Psychological Sociology 6 weeks 5 hours per week
Social Viewpoints and Attitudes 6 weeks 5 hours per week
Criminology 12 weeks 2 hours per week

 

History

At the University of Colorado, 1919-1921
Colonization of North America 24 weeks 2 hours per week
The Westward Movement 6 weeks 5 hours per week

 

Education

At the University of Colorado, 1919-1921
History and Philosophy of Education 24 weeks 3 hours per week
Seminar in Education 24 weeks 2 hours per week

 

Political Science

At the University of Colorado, 1919-1921
Municipal Functions and Problems 12 weeks 3 hours per week
International Law 12 weeks 3 hours per week
World Govt. and Politics 6 weeks 5 hours per week
Political Parties and Party Problems 24 weeks 2 hours per week

 

Summary

Majors

Work in Political Economy at other institutions, certified by the Department of Political Economy of the University of Chicago as equivalent to…
Work in Political Economy at the University of Chicago… 9
Work in Sociology at other institutions, certified by the Dept. of Sociology of the Univ. of Chicago as equiv. to …
Work in History at other institutions, certified by the Dept. of History of the Univ. of Chicago as equiv. to…
Work in Education at other institutions, certified by the School of Education of the Univ. of Chicago as equiv. to… 2
Work in Pol. Science at other institutions, certified by the Dept. of Pol. Science of the Univ. of Chicago as equiv. to… 3
Total majors in Political Economy… 17½
Total majors in other subjects… 9
Grand Total… 26½

 

For the past four years I have been a member of the faculty of the Department of Economics of the State College of Washington, for the past three years with the rank of assistant professor of economics. During this time I have taught the following subjects, having given courses in all of these subjects several times: (1) Economic Geography; (2) Foreign Trade; (3) Railway Transportation; (4) Agricultural Economics; (5) Marketing Farm Products; (6) Co-operative Marketing of Farm Products; (7) Money and Banking; (8) Principles of Economics, elementary and intermediate courses.

For the spring quarter I am planning to register for Political Economy 303, Modern Tendencies in Economics, to continue the research work on my thesis subject, and if advised to do so to register for one additional course. I do not expect to be able to complete the thesis by the close of the spring quarter, but am trusting that I may be able to meet all course requirements and to complete the thesis and take the thesis examination before the close of the summer quarter.

It appears evident that my course requirements are dependent upon the amount of work in allied departments, consisting of courses already completed in other institutions, which will be approved by the Department as a part of the preparation for the degree. I am submitting this statement in the hope that I may have from the Department at an early date definite notification of the courses which I shall have yet to complete in fulfillment of the requirements for the degree.

Certified transcripts of records of courses completed at other institutions and of the valuations placed upon this work by the various departments of the University of Chicago, as enumerated in this communication, are on file in the office of the Deans of the Graduate Schools.

Respectfully yours,
[signed]
Edwin F. Dummeier

________________________

Dummeier proposing his doctoral thesis subject

5757 University Avenue, Chicago, Illinois,
January 21, 1926

Professor L.C. Marshall, Chairman,
Department of Political Economy
The University of Chicago.

Dear Sir:

I am hereby presenting for your approval the subject and a brief prospectus of the thesis which I propose later to submit in partial fulfillment of the requirements for the degree of Doctor of Philosophy in Political Economy. The subject of the proposed thesis is “The Marketing of Pacific Coast Fruits in Chicago”.

While the prospectus is designed to give some idea of the general nature of the proposed study, it does not indicate the degrees of relative intensity with which it is proposed to treat the various phases of the general subject. All phases will be treated to the extent of critically surveying the existing literature pertaining to them and making some supplementary field study. But the study as a whole will be based not on existing literature, but on original field observations and a study of commercial records. As an exhaustive study of all phases of proposed subject by these methods is beyond the capacity of any one individual it is proposed to investigate with much more detail some phases than others. The degree with which this specialization will be devoted to particular ones of the subheads listed in the outline will depend in part upon the degree of cooperation received from the trade and, therefore cannot be definitely stated in advance. Representative, however, as a phase of the general subject in regard to which there is at the present time only the most meager published information and which may be studied is the fruit and vegetable auction as a marketing institution. As the auction is mostly used in connection with the marketing of Pacific Coast products this would be a natural subdivision of the main subject.

The whole study has as its primary object the evaluation of existing methods in regard to these products as to their social efficiency and social significance.

Yours respectfully,
[signed]
E. F. Dummeier

Thesis
THE MARKETING OF PACIFIC COAST FRUITS IN CHICAGO

Chapter

  1. Introduction
    1. The importance of the study
    2. Method of treatment
      1. Emphasis on a few commodities, especially apples
      2. Emphasis on change and development in marketing methods
    3. Specific objectives
      1. Primary objective: To evaluate comparative merits of different methods of performing marketing services.
      2. Secondary objectives: To show the relation of Chicago to the producing areas; to describe physical facilities of the market and the physical movements of these products thru the market; to determine costs of marketing these products and reasons for these costs; to examine factors influencing demand and to examine trends of change and their causes.
  2. Chicago and the Regions of Supply
    1. Data on production, arrivals, and unloads at Chicago. Data on storage movements and reshipments from Chicago.
    2. The historical development of the industry, its present status, and its current trends.
  3. The Physical Facilities of the Market and Physical Commodity Movements
    1. Transportation services and facilities
    2. Wholesale receiving
    3. Auctions
    4. Peddlers
    5. Retailers
  4. Carload Distributors, Brokers, and Carload Receivers
    1. Numbers and classes of dealers
    2. Marketing services performed and trade practices
    3. Charges for services
  5.  Auctions
    1. Extent of movement thru auctions
    2. Auction methods
    3. Auction charges
  6. Jobbers and Shippers
    1. Numbers and classes of dealers
    2. Methods of buying and selling
    3. Margins and costs
  7.  Retailers
    1. Numbers and classes of dealers
    2. Methods of buying and selling
    3. Margins and costs
  8. Marketing Costs
    1. Critical consideration of marketing costs, especially of oranges and apples, on the basis of differences in marketing methods employed until time of sale to jobbers.
    2. Particular consideration of the desirability of selling at auction.
  9. Marketing Costs (Continued)
    1. Critical consideration of marketing costs subsequent to time of sale to jobbers
  10. Factors Influencing Demand
  11. Summary and General Conclusions

________________________

Department approves Dummeier’s thesis subject

January 27, 1926

Mr. E. F. Dummeier
5757 University Avenue
Chicago, Illinois

My dear Mr. Dummeier:

The Department of Political Economy accepts as your thesis subject “The Marketing of Pacific Coast Fruits in Chicago.”

It is our understanding that you will carry on work in connection with this thesis under Mr. Duddy.

Yours very sincerely,
[Unsigned copy, L.C. Marshall]

LCM:MLH

________________________

Department Head Marshall asks his colleague to double-check the Dummeier transcripts for possible feedback

The University of Chicago
Department of Political Economy
February 1, 1926

Mr. C. W. Wright
University of Chicago
Faculty Exchange

My dear Mr. Wright:

I enclose a letter from Mr. Dummeier. I have written him concerning the field “Transportation and Communication.” Perhaps you will wish to look over his statement of courses and credits to see if any action needs to be taken concerning them.

Yours very sincerely,
[signed]
L.C. Marshall

LCM:MLH
Enclosure

________________________

The University of Chicago
Department of Political Economy

Edwin F. Dummeier

A. B. University of Louisiana, 1918
A. M. University of Colorado, 1921

Summer Quarter, 1925

Pol Econ. 334 A
C & A 388 A
C & A 388B A

French and German Exams. Passed. Sept. 1, 1925

Grad. Work in other insti. September 1, 1925

University of Colorado
Soc. (Faris) 2½ majors
Residence credit 1 Quarter

Grad. work in other insti. September 3, 1925

University of Colorado
Pol. Econ. (Field) 5½ majors
Residence credit 2 Quarters

 

University of California and Wisconsin
Pol. Econ. (Field) 3 majors
Residence credit 1 Quarter

 

Autumn Quarter, 1925

Pol Econ. 301 A
C & A 313 [blank]
C & A 345 A
C & A 385 A
C & A 386 A

 

Grad. work in other insti. Jan 4, 1925

University of Colorado
Educ. (C.H. Judd) 2
Pol. Sci. (C.E. Merriam) 3
Residence Credit 1 Quarter
History (C.F. Huth) 1 ½
Residence Credit ½ Quarter

________________________

Department requests clarification regarding the proposed field “Transportation and Communication”

February 1, 1926

Mr. E. F. Dummeier
5757 University Avenue
Chicago, Illinois

My dear Mr. Dummeier:

It seems entirely probable that the Department will approve the four fields suggested in your letter of January 21st.

The Department has, however, asked me to secure from you a more detailed statement of your understanding of the territory that would be covered by the field “Transportation and Communication.”

Yours very sincerely,

[Unsigned: L. C. Marshall]

LCM:MLH

________________________

Schedule of written field examinations

February 2, 1926

Mr. E. F. Dummeier
5757 University Avenue
Chicago, Illinois

My dear Mr. Dummeier:

This is just to let you know that I have you scheduled to take the following examinations on the dates mentioned.

February 13, Economic Theory. 8:30 A.M.

February 20th, Pecuniary and Financial Systems, 8:30 A.M.

February 27th, Transportation and Communication 8:30 A.M.

The questions will be given out at Harper E 57. Please let me know at once if the above schedule is incorrect.

Yours very sincerely,
[Unsigned copy: Margaret McKugo]

MM:MLH

________________________

Dummeier clarifies his understanding of the field “Transportation and Communication”

5757 University Avenue, Chicago, Illinois,
February 4, 1926

Professor L.C. Marshall, Chairman,
Department of Political Economy
The University of Chicago.

Dear Sir:

In reply to your letter of February 1st I am hereby submitting the following as my understanding of the territory that would be covered by the field “Transportation and Communication”, which was proposed by me as one of my fields of specialization in my candidacy for the Ph.D. degree.

As to agencies, I understand the field to include all the agencies of land and water transportation. Major emphasis should, however, be placed upon railway transportation in the United States. Agencies supplying communication other than physical transportation would include the telephone and telegraph. As compared with railway transportation these are of less importance, and as they present relatively few distinctive problems they may be said to be somewhat incidental to the main field.

With regard to the above mentioned agencies consideration should be given to phenomena and problems of the character of those with which Political Economy in general concerns itself. These should include the following:

  1. The historical development of the various transportation agencies,
  2. The services performed and economic significance of the various agencies,
  3. Theories of rate making, particularly railway rates,
  4. Rate making practices and rate systems,
  5. Railroad finance,
  6. Sufficient knowledge of the technic of operation to be able to consider intelligently questions of public policy with regard to railroads and other transportation agencies,
  7. The economic and legal bases of the regulation of public carriers and the history of their public promotion and regulation,
  8. Various present day transportation problems in which the general public has an interest, such as valuation, consolidation, and government ownership or operation.

The above indicates the general scope and to some extent the relative emphasis of the constituent parts of the field of Transportation and Communication as a field of Political Economy as I understand it.

Most respectfully yours,
[signed]
Edwin F. Dummeier

________________________

Wright’s Response to Marshall’s Feb. 1, 1926 Inquiry

THE UNIVERSITY OF CHICAGO
The School of Commerce and Administration

Memorandum to Marshall from Wright
[no date, but probably early Feb. 1926]

After surveying Mr. Dummeier’s record of courses taken, it seems to me that in the four fields chosen he has not covered the following.

Theory: History of Theory. Only partly covered.

Unsettled Problems. He plans to take this in the Spring.

Marketing: Advertising. I am not certain as to this.

Transportation: Public Control of Railroads.

Of the specific general requirement he has covered Statistics and Accounting but not Economic History of the U.S. I gathered from the discussion at the Dept. meeting that the members of the Department would refuse to tell him specific courses that were required, though personally I do not consider this a reasonable attitude.

C.W.W.

________________________

Response of Department to Dummeier’s follow-up regarding his examination field “Transportation and Communication”

March 2, 1926

Mr. E. F. Dummeier
5737 University Avenue
Chicago, Illinois

My dear Mr. Dummeier:

I spoke to Mr. Wright and he told me that your recommendation had come before the Department, but he could not at this time give you a written statement concerning it. He is turning your letter over to Mr. Marshall who will write you as soon as he returns to the office.

Yours very sincerely,
[Unsigned copy: Margaret McKugo]

MM:MLH

________________________

Economics Department Record of Dummeier’s Written Ph.D. Examination Grades
(First attempt)

Winter Qr. 1926

E. F. Dummeier

Economic Theory

Viner — Pass Fair
Clark — B

Pec. And Fin. Sys.

Mints — Failed
Wright — C
Meech — Failed

Trans. & Com.

Clark — Passed
Sorrell — [Blank]
Duddy — Passed

________________________

Department’s decisions
regarding credits recognized
plus advice on “possible gaps”

March 16, 1926

Mr. E. F. Dummeier
5757 University Avenue
Chicago, Illinois

My dear Mr. Dummeier:

After examining your credits as officially certified by various departmental representatives it seems clear that you have met the general requirements as far as the total number of majors is concerned.

The only issues outstanding are these:

  1. There is a requirement that a candidate for the doctor’s degree shall have covered work in the Economic History of the United States. I am uncertain whether you have taken care of this requirement.
  2. You will, of course, need to be prepared to pass the examinations in four fields. As you know no specific courses are required in connection with these examinations. The candidate is expected to work up each field in a rather comprehensive way.

Certain questions arise in my mind with respect to these examinations. Have you prepared yourself in the field of Public Control of Railroads? Have you done so in the general field of Advertising? Have you done so in the History of Economic Thought? You will, I am sure, realize that these inquiries do not indicate the necessity of your taking specific courses in these territories. I mention them merely as possible gaps in your thinking in these fields.

Yours very sincerely,
[Unsigned copy: L. C. Marshall]

LCM:MLH

________________________

Dummeier informed that he passed two of his three written examinations
[Carbon copy]

March 24, 1926

Mr. E. F. Dummeier
5757 University Avenue
Chicago, Illinois

My dear Mr. Dummeier:

The final reports for the written examinations taken by you during the Winter Quarter, 1926 in partial satisfaction for the degree of Doctor of Philosophy are as follows:

Economic Theory — Passed

Pecuniary and Financial System — Failed

Transportation and Communication — Passed

Yours very sincerely,
[Unsigned copy: L. C. Marshall]

LCM:MLH

________________________

Economics Department Record of Dummeier’s Written Examination Grades
(Second attempt: Pecuniary and Financial Systems Field)

Pecuniary and Financial Systems

Mints — Pass
Cox — Pass

________________________

Dummeier told he successfully passes his third written examinations
[Carbon copy]

June 8, 1926

Mr. E. F. Dummeier
5757 University Avenue
Chicago, Illinois

My dear Mr. Dummeier:

I am pleased to report that you have passed the Pecuniary and Financial System examination, taken in the Spring Quarter, 1926, in partial satisfaction for the degree of Doctor of Philosophy.

Yours very sincerely,
[Unsigned copy: L. C. Marshall]

LCM:MLH

________________________

Dummeier’s Principal Advisor not in Chicago during the summer quarter (when the thesis is expected to be completed and submitted)

The University of Chicago
Local Community Research Committee
Address: Faculty Exchange. The University of Chicago

June 7, 1926

Mr. L.C. Marshall, Dean
Department of Political Economy
University of Chicago

Dear Mr. Marshall:

My absence during the Summer Quarter means that some one must supervise the students who have been working under me in community research. Mr. Dummeier, who plans to get his degree in Political Economy, is quite well along with his work and I should like to recommend that either Mr. Wright or Mr. Viner look after him. He is going to develop a section on price study and Viner would be a help there.

The other men, Davidson, Journey and Weaver, are planning to come up in Commerce and Administration, and I am making recommendations to Mr. Spencer to take care of them. In the case of all of these men, I shall want to read copies of their theses as they come in. Both Mr. Dummeier and Mr. Journey have their outlines fully developed and have begun to write.

Yours very truly,

[signed]
E.A. Duddy

EAD:JS

________________________

Department Head Marshall turns to Jacob Viner
for last-minute thesis advice

June 8, 1926

[Memorandum to:] Jacob Viner

[From:] L. C. Marshall

Mr. Dummeier has been working with Mr. Duddy, but Mr. Duddy is to be away this coming summer. I wonder if you would be willing to look after Mr. Dummeier’s work on the thesis since he is planning to develop a section on price study.

The matter is one upon which the Department needs to take action in view of the fact that Mr. Dummeier plans to take his degree in Political Economy.

LCM:MLH

_______________________

Viner “gratefully” accepts the “chore”

The University of Chicago
Department of Political Economy

June 10, 1926

Mr. L. C. Marshall
Faculty Exchange

My dear Mr. Marshall:

You may send on Mr. Dummeier to me. I will take over the job of supervision of his research during Mr. Duddy’s absence, inasmuch as I have been unable to think up a good excuse for evading the chore.

Gratefully yours,
[signed]
Jacob Viner

_______________________

Notification that Viner Will Serve as Substitute Research Supervisor

June 17, 1926

Mr. E. F. Dummeier
5757 University Avenue
Chicago, Illinois

My dear Mr. Dummeier:

I have had a note from Mr. Viner indicating his willingness to supervise your research in Mr. Duddy’s absence.

Yours very sincerely,
[Unsigned copy: L. C. Marshall]

LCM:MLH

________________________

Official Examination Notice for E. F. Dummeier
(with Prof. Meech’s scribbled note that he will be unable to attend)

________________________

COURSES PRESENTED BY EDWIN F. DUMMEIER
FOR THE DEGREE Ph.D. IN ECONOMICS
AT THE UNIVERSITY OF CHICAGO

Majors
Pol. Econ. 334 Money and Prices. Hardy 1
C & A 388 B Marketing Farm Products, Weld 1
C & A 388 A Cooperative Farm Marketing. Jesness 1
Pol. Econ. 301 Neo-Classical Economics. Viner 1
C & A 345. Personnel Administration. Stone 1
C & A 386 Terminal Marketing Research. Duddy 1
C & A 355 Business Finance and Investment. Meech 1
C & A 375 Business Forecasting. Cox 1
Pol. Econ. 499 Terminal Marketing Research Duddy 3
Pol. Econ. 499 Terminal Marketing Research. Viner 3
TOTAL 14

Graduate Work at Other Institutions

Economics
Transportation. Principles Univ. of Cal. Dixon
Transportation. Current Problem[s]. Univ. of Cal. Dixon
Pacific Coast Rate Problems. Univ. of Cal. Harraman
Farmer Movements. Univ. of Wis. Hibbard
The Classical Economists. Univ. of Wis. Scott
Statistics. Univ. of Wis. Lescohier
Money and Banking. Univ. of Colo. Ingram
Taxation. Univ. of Colo. Ingram
Immigration. Univ. of Colo. Ingram
Business Organization. Univ. of Colo. Ingram
Seminar in Economics. Univ. of Colo. Bushee
Thesis “Financing Public Education in Colorado.”
Total (Field)
Economics Total   22½

 

 

Education Total Judd 2
Sociology Total Faris
Political Science Total Merriam 3
History Total Huth
Grand Total   31½

 

________________________

Memo from Millis announcing/reminding about oral examination date
[Carbon copy]

The University of Chicago
The Department of Political Economy

August 17, 1926

Memorandum to:

N. W. Barnes [Associate Professor of Marketing]
P. A. Douglas [Associate Professor of Industrial Relations]
L. H. Grinstead [Visiting Assistant Professor from Ohio State University]
G. G. Huebner [Visiting Professor from the U. of Pennsylvania]
L. C. Sorrell [Assistant Professor of Transportation and Communication]
Jacob Viner [Professor of Political Economy]
C. W. Wright [Professor of Political Economy]

From: H. A. Millis

This is just to let you know that E. F. Dummeier will come up for his oral examination on Monday, August 23, at 3 o’clock in Harper E 57.

If it is impossible for you to be present will you please notify Miss McKugo in Harper E 57?

________________________

Memo from Millis announcing/reminding about oral examination date
[Carbon copy]

[Memorandum To:] L. S. Lyon [Visiting Professor from Robert Brookings Graduate School of Economics and Government]

[From: H. A. Millis]

August 18, 1926

This is just to let you know that E. F. Dummeier will come up for his oral examination on Monday, August 23, at 3 o’clock in Harper E 57.

If it is impossible for you to be present will you please notify Miss McKugo in Harper E 57?

________________________

A “Thank-you” to Marshall for his support
Note: Dunnmeier’s article on auctions apparently never published

 

The State College of Washington
Pullman, Washington
Department of Business Administration

December 28, 1926.

Professor Leon C. Marshall
Department of Economics
University of Chicago
Chicago, Illinois

Dear Professor Marshall:

I am enclosing herewith a review of Benton’s “Marketing of Farm Products” for the Journal of Political Economy. I had hoped to have gotten this review to you at an earlier date, but teaching duties have kept me so busy as to delay its completion somewhat longer than I anticipated.

Not long ago I received a letter from professor Duddy, in which he stated that you had spoken to him with regard to my writing an article for the Journal on the fruit auction as a marketing agency, the article to be based on my first hand research work in Chicago. I have started the preparation of such an article and hope to submit it within the very near future.

I have found on my return to my duties here that my year at the University of Chicago has been of very large benefit to me, and I continue to feel most grateful to you for your part in making that year possible.

Most cordially yours,
[signed]
E. F. Dummeier

EFD/EIB

Source:  University of Chicago Archives. Economics Department. Records & Addenda. Box 6, Folder 12.

Image: “Dummeier Rites Are Held Today,” Spokane Chronicle, June 18, 1946.

Categories
Chicago Exam Questions

Chicago. Price Theory (B). Final Exam Questions. Friedman, December 1959

 

Along with the exam questions and answers transcribed below, Milton Friedman’s papers include the official course registration list together with his hand-written grades that were calculated based on the results of this exam and a problem set. During the autumn quarter of 1959 thirty students were enrolled in Friedman’s course with two students receiving incompletes. For the remaining 28 students 2 A’s, 10 B’s, 10 C’s, 5 D’s and 1 F grades were awarded. The two A grades went to Arthur Lionel Broida (1963 economics Ph.D. “Liqudity as a Variable in Monetary Analysis”) and Charles N. Tingley [probably Charles Nicholas Tingley, Yale 1957. Worked for Humble Oil and Refining Company  at the time of his marriage to Cary Clift MacFadden in 1964].

______________________________

ECONOMICS 301
[Price Theory B]
Final Examination
December 16, 1959

  1. [30 points total] The accompanying graph gives a set of consumption indifference curves for two commodities or services each of which for some range of quantities and in combination with some amounts of the other is capable of being either a “good” or a “bad” (a “product” or a “factor”) like books and bookshelves, or cutting the grass and playing the piano (either of which may be “labor” or “play”). Of the curves drawn, I1 corresponds to the lowest level of utility. Answer this question on this paper, wherever relevant filling in the blanks.

    1. [3 points] What is the interpretation to be placed on point B? [Answer: Bliss]
    2. [8 points total; 2 points each] Mark of I1 into four sectors [Answers circled on figure] according as

(1) _____both X and Y are “goods”
(2) _____X is a “good” and Y is a “bad”
(3) _____X is a “bad” and Y is a “good”
(4) _____X and Y are both “bads”

Use letters to designate the dividing points between the sectors and enter the description of each sector in the proper place above. [Answer: see X’s used in figure]

    1. Budget lines AC and A´C´ are the usual type which supposes that the consumer must pay for both products and has a fixed sum to spend on both.

(1) [1 point] The consumer’s optimum position for A´C´ is [Answer: D´].
(2) [4 points] The consumer’s optimum position for AC is [Answer: B].

    1. On budget lines EF and GH, one of the commodities is something the consumer must pay for (it is a “product” and has a positive price), the other is something he gets paid to accept (it is a “factor” and has a negative price). In addition for both lines, the consumer has a fixed sum derived from some other source to spend.

(1) [2 points] For EF [Answer: Y] is the product; [Answer: X] is the factor.
(2) [2 points] For GH [Answer: X] is the product; [Answer: Y] is the factor.

    1. For OK also one commodity is a product and one is a factor but there is no additional source of expenditures and hence no way from the line itself to know which is which. However, it does make a difference to the optimum position which is which.

(1) [2 points] If X is a factor and Y a product, then the optimum point is [Answer: B].
(2) [2 points] If Y is a factor and X a product, then the optimum point is [Answer: K].
(3) [5 points] Can you suggest a simple graphical way of distinguishing the two cases? [Answer: Shading areas].

  1. [30 points] Find the mistakes (there are at least six) in the accompanying diagram showing long and short run marginal and average cost curves, and explain the general principle corresponding to each particular mistake.

[Answers: (1)SRMC ≠ SRAC at minimum; (2) SRAC < LRAC; (3) SRMC ≠ LRMC where SRAC = LRAC; (4) SRMC < SRAC when SRAC rising; (5) SRMC < LRMC when to left of point of tangency of SRAC and LRAC; (6) LRAC> LRMC when LRAC rising or LRMC ≠ LRAC when LRAC max).]

  1. [24 points] Define briefly the following terms [3 points each]:
    1. Marginal revenue
    2. Fixed cost
    3. Income elasticity
    4. Profit
    5. Production function
    6. Diminishing returns
    7. Inferior good
    8. Luxury
  2. [30 points] Discuss the following quotations:
    1. (from a newspaper story) “The Sun Rose Bar and Grill…advertised ‘the largest glass of beer in the city for five cents’ and did a tremendous business in eight-ounce glasses of beer as soon as the public realized it was no April Fool proposition…
      ‘If enough of us do this’ said…one of the proprietors, ‘the brewers will have to cut prices!’”.
    2. (from a newspaper story) “Domestic producers of oil contend that unrestricted imports hurt them not only because they swell the supply, but because a barrel of foreign crude costs about $1 less than a comparable barrel of U.S. crude.”
    3. “All of this is to say, of course, that in practice what we have to reckon with is not a unique marginal cost for a given level of output, but a complex of marginal costs, each of which is pertinent to a particular period of time. As a longer period of time is considered, more of the ‘fixed factors’ become variable. Because of this greater flexibility in the production process, long-run marginal cost will generally be less than short-run marginal costs.” A. Bergson in A Survey of Contemporary Economics.

 

Source: Hoover Institution Archives. Milton Friedman Papers, Box 77, Folder 2: “University of Chicago, Econ. 301”.

Image Source: Detail from picture of Milton Friedman at the Center for Advanced Study in the Behavioral Sciences, Stanford. University of Chicago Photographic Archive, pf1-06234, Hanna Holborn Gray Special Collections Research Center, University of Chicago Library.